SlideShare a Scribd company logo
1 of 10
Download to read offline
1121
PRINCIPLES
Background
The appendix was once considered a vestigial organ; however, it
is currently theorized that it serves as a repository for commensal
bacteria that assist in normal digestive processes and may allow
for recolonization of intestinal flora in times of enteric bacterial
destruction. Phylogenetic studies have supported the appendix as
likely having a so-called positive fitness value during mammalian
evolution, whereas recent clinical research studies have demon-
strated a possible increased risk of clostridial infections in patients
who have had prior appendectomies.1-3
Appendicitis is the most common cause of acute abdomi-
nal pain requiring operative intervention in patients younger
than 50 years. It is the most common nonobstetric abdominal
emergency in pregnant females, usually occurring in the second
trimester. Risk factors for appendicitis include white ethnicity,
male gender, and young age (69% of cases occur in patients <30
years). Although males have an increased risk of having appen-
dicitis (1.4 : 1), females have an almost twofold increased risk of
undergoing an appendectomy, which is partially related to the fact
that women have gynecologic conditions that frequently mimic
appendicitis.
Anatomy and Physiology
The vermiform appendix is a blind-ended tube that originates
from the cecum, approximately 3 cm from the ileocecal valve. It
is considered part of the cecum and has the same histologic
arrangement as the large intestine. A unique aspect of the appen-
dix is the large masses of lymphoid tissue in the mucosa and
submucosa. Although it has an average length of 8 to 10 cm, the
appendix may be more than 20 cm in length, thereby allowing it
to traverse into the left lower or right upper quadrants of the
abdomen. The average diameter of the appendix ranges from 6 to
11 mm; thus, appendiceal diameter alone, in the absence of other
radiologic findings, often does not imply appendicitis. Finally, a
normal appendix has an average wall thickness of 1.5 mm.
The appendix maintains afferent sensory fibers that follow the
sympathetic innervation and enter the spinal cord at the 10th
thoracic level (T10). It is these nerves that carry the sensation of
pain and result in the periumbilical discomfort associated with
early appendicitis.
There are three important anatomic features of the appendix
that determine the site of the patient’s pain and tenderness when
the organ is inflamed: (1) the location of the origin of the appen-
dix off the cecum; (2) the course the appendix takes from this
origin; and (3) the length of the appendix (as described above;
Fig. 83.1). All these features are variable, resulting in a wide range
of symptoms and signs, which often creates difficulty with the
clinical diagnosis of appendicitis. Although the location of the
origin of the appendix is generally thought to be positioned at
McBurney’s point (exactly between an 1.5 to 2 inches from the
right anterior superior iliac spine on a straight line drawn to the
umbilicus), the base may be as much as 10 cm away from this site.
In fact, only 40% of patients have the base of their appendix
within 3m of McBurney’s point, with 36% of patients having the
base more than 5m away. Next, there is significant variation
involving the course of the appendix from its cecal origin.
The frequency with which the appendix is found in various loca-
tions is illustrated in Fig. 83.2. Finally, in rare cases, patients
may present with left lower quadrant pain or right flank pain
due to a very long appendix traveling into the left lower abdomen
or a retrocecal location extending into the retroperitoneum,
respectively.
Pathophysiology
The cause of appendicitis is rooted in obstruction of the appen-
diceal lumen. The underlying pathophysiology is progressive in
nature and best understood in a stepwise fashion—appendiceal
obstruction prevents egress of mucus and bacteria from the
appendix; continued mucous production and bacterial prolifera-
tion result in luminal distention, which stimulates the T10 visceral
afferent nerves, creating periumbilical pain typically lasting 4 to
6 hours. Intraluminal pressure eventually exceeds local capillary
pressure in the appendiceal wall, preventing arterial perfusion and
resulting in tissue ischemia and inflammation; ischemia and
inflammation compromise the integrity of the appendiceal wall
bacteria and then invade the appendiceal wall. This causes trans-
mural inflammation that extends into the surrounding tissues
(peritoneal, ileocecal, and pelvic areas), resulting in somatic local-
ized pain, typically focused in the right lower quadrant. If this
process continues, the appendix becomes necrotic and perforates,
releasing enteric contents into the peritoneum and resulting in
peritonitis and, typically, diffuse abdominal pain. The length
of time from the onset of symptoms to perforation is highly
variable.
Although obstruction of the appendiceal lumen is thought to
be a common inciting factor in appendicitis, the cause of this
obstruction is variable and, in many cases, a source of obstruction
is not discovered on imaging or pathology. Fecaliths (hard stools)
are the most common cause of obstruction in nonperforated
appendicitis (65%), followed by appendicoliths (calcified depos-
its) and lymphoid hyperplasia (primary or secondary to an enteric
infection). Other causes of obstruction are rare; these include fecal
stasis, foreign bodies (eg, vegetable matter, inspissated barium),
tumors, and intestinal parasites.
There are two additional pathologic processes associated
with appendicitis. First, so-called tip appendicitis is appendiceal
inflammation localized to the distal end of the appendix. The
clinical significance of this disorder is that it may be missed on
imaging due to the limited extent of disease and the lack of classic
findings associated with appendicitis; it has been demonstrated to
contribute to the false-negative rate associated with computed
tomography (CT) imaging. Second, stump appendicitis is a very
rare entity that results from inflammation of the appendiceal
remnant that may persist after the appendix has been removed
surgically. Timing of its occurrence varies; it has been diagnosed
C H A P T E R 83 
Acute Appendicitis
Michael Alan Cole  |  Robert David Huang
Descargado para Francisco Medina Anderson (fz.medina@hotmail.com) en Organización de Estados Iberoamericanos - Remote Access de ClinicalKey.es por Elsevier en julio 07, 2017.
Para uso personal exclusivamente. No se permiten otros usos sin autorización. Copyright ©2017. Elsevier Inc. Todos los derechos reservados.
1122 PART III  Medicine and Surgery  |  SECTION Five  Gastrointestinal System
treatment, and disposition. This approach should focus on the
history and physical examination findings that have the greatest
predictive values.
History
The history and review of systems provide insight into the patient’s
symptoms and help determine alternative diagnoses. A prior
history of similar symptoms suggests an alternative diagnosis,
because appendicitis is an acute illness. Furthermore, because the
pathophysiology of appendicitis is a progressive process,a patient’s
symptoms typically worsen over the course of the illness until
perforation of the appendix occurs. At this point, the patient may
receive some temporary relief due to a decrease of intraluminal
pressure, but will subsequently become very ill from the resultant
peritonitis. See Table 83.1 for a list common symptoms and their
value in predicting the likelihood of appendicitis. Duration of
symptoms is variable and often is not useful in the assessment of
appendicitis.
When considering features that exclude appendicitis, findings
that decrease the likelihood of appendicitis include absence of
RLQ pain and a history of similar pain in the past. Nevertheless,
these findings in isolation do not exclude appendicitis as a pos-
sible diagnosis and are best used as part of a comprehensive clini-
cal decision making process.
Physical Examination
All patients with abdominal pain should be fully disrobed, and
female patients should ideally be placed in a room in which a
pelvic examination can be performed (see Table 83.1). Classic,
eponymous examination maneuvers for appendicitis have overall
poor sensitivity but, if present, have a modest predictive value
(Table 83.2). Although McBurney’s point tenderness has a low
correlation with appendiceal location and is not highly sensitive
for appendicitis, tenderness at this location does have a modest
predictive value for appendicitis.
A genitourinary examination should be performed to assess
for testicular pathology or hernias in males and pelvic pathology
in females. Cervical motion tenderness (CMT) is not specific for
pelvic pathology and is noted in 28% of female patient with
appendicitis. A rectal examination contributes little toward the
assessment of appendicitis and is not routinely recommended.6
DIFFERENTIAL DIAGNOSIS
There are many diagnoses that mimic appendicitis; alternatively,
appendicitis may present atypically and must be considered in all
between 4 days and up to 50 years after the initial
appendectomy.4
CLINICAL FEATURES
Typical presentations of appendicitis often allow an experienced
provider to hone in on a diagnosis in a rather rapid fashion;
unfortunately, typical findings are the exception and not the rule.
For historical purposes, the constellation findings classically asso-
ciated with acute appendicitis include younger age, epigastric pain
that migrates to the right lower quadrant (RLQ), and tenderness
of the RLQ. However, the combination of these findings occurs in
less than 50% of patients with acute appendicitis thus limiting
their use in clinical decision making.4a
Missed acute appendicitis
is one of the most common causes of litigation surrounding emer-
gency medicine, which speaks to the challenges in diagnosing this
illness in the presence of often ambiguous symptoms.5
No one element of the history or physical examination can
reliably be used to diagnose or exclude appendicitis. Therefore,
focusing on a single finding may lead to misdiagnosis. Rather, a
comprehensive approach using multiple elements of the history,
physical examination, and laboratory data should be used to risk-
stratify patients to make informed decisions regarding imaging,
Fig. 83.1.  This figure demonstrates the variation in location of the base
of the appendix and its course within the peritoneum. McBurney’s point
classically represents the point of maximal tenderness in appendicitis;
however, depending on the path, length, and degree of inflammation of
the appendix, the true point of maximal tenderness is highly variable.
Subcecal (2%)
Pelvic (32%)
Preilial (1%)
Postileal (0.4%)
Retrocecal
(64%)
The origin of the
appendix can vary as
much as 10 cm from
McBurney’s point.
TABLE 83.1 
Predictive Value of Common Symptoms and Signs in Diagnosing Appendicitis
FEATURE MODERATELY USEFUL MILDLY USEFUL NOT USEFUL
Historical features RLQ pain
Migration of pain to the RLQ
Presence of pain prior to vomiting
No history of prior similar pain
Vomiting
Male gender
Pain worsened when driving over
speed bumps34
Anorexia
Nausea
Pain worse with cough or movement
Physical examination features35
RLQ tenderness
Abdominal wall rigidity
Pain focused at McBurney’s point
Rebound tenderness
Guarding
Temperature > 38.3°C (101°F)
Percussion tenderness
Psoas sign
Rectal examination
Increased skin temperature
RLQ, Right lower quadrant.
Adapted from Laurell H, Hansson L-E, Gunnarsson U. Manifestations of acute appendicitis: a prospective study on acute abdominal pain. Dig Surg 30:198–206, 2013.
Descargado para Francisco Medina Anderson (fz.medina@hotmail.com) en Organización de Estados Iberoamericanos - Remote Access de ClinicalKey.es por Elsevier en julio 07, 2017.
Para uso personal exclusivamente. No se permiten otros usos sin autorización. Copyright ©2017. Elsevier Inc. Todos los derechos reservados.
1123CHAPTER 83  Acute Appendicitis
comprehensive assessment of the patient’s condition and further
risk-stratify the patient for treatment and disposition purposes.
White Blood Cell Count
A patient’s white blood cell (WBC) count does not by itself have
the sensitivity, specificity, or predictive value necessary to be clini-
cally useful in diagnosing or excluding appendicitis. An elevated
WBC count (>10,000–12,000/mm3
) has a sensitivity of 62% to
85%, specificity of 32% to 82%, positive LR of 1.59 to 2.7, and
negative LR of 0.25 to 0.46. Even in a subgroup analysis of
patients presenting with abdominal pain, not just those with right
lower quadrant pain. Table 83.3 lists the most common differen-
tial diagnoses for appendicitis.
DIAGNOSTIC TESTING
Laboratory Data
Laboratory data should not be viewed as diagnostic for appendi-
citis. Rather, it should be used in association with the patient’s
clinical history and physical examination to formulate a more
Fig. 83.2.  Suggested clinical management pathway for emergency department patients with possible
appendicitis. Gen., General; OB, obstetrician.
Develop pretest probability
for appendicitis based on history,
physical examination, and
laboratory data
Treat symptomatically
Low risk
• Consider/treat
alternative
diagnoses
• Discharge with
precautionary
instructions
High riskPregnant
No
Negative/
nondiagnostic Positive Positive
Negative/
nondiagnostic
Positive
Negative Negative
Positive
Yes Consult OB &
Gen. Surgery
Moderate risk*
No Yes
No Yes
Operative
appendix
removal within
12 hours of
diagnosis
Admit for
continued
IV antibiotics,
serial examinations
and observation
NoYes
CT with IV contrast
(no enteric contrast)
Is the patient still
symptomatic
Diagnosis of
appendicitis is made
Administer IV
antibiotics
Admit for
observation
and
symptomatic
therapy
• Consider/treat
alternative
diagnoses
• Discharge with
precautionary
instructions
Admit for
observation
and
symptomatic
therapy
Surgical
consultation
Is the patient a
candidate for
conservative
management
Is the patient still
symptomatic?
MRI with no IV contrast
(+/- enteric contrast)
Consider graded
compression
ultrasound**
Graded
compression
ultrasound
*In moderate pre-test probability patients, the
provider may consider admission for serial
examinations or discharge in select cases.
**In pediatric patients, graded compression
ultrasound should always be the first imaging
test performed.
Descargado para Francisco Medina Anderson (fz.medina@hotmail.com) en Organización de Estados Iberoamericanos - Remote Access de ClinicalKey.es por Elsevier en julio 07, 2017.
Para uso personal exclusivamente. No se permiten otros usos sin autorización. Copyright ©2017. Elsevier Inc. Todos los derechos reservados.
1124 PART III  Medicine and Surgery  |  SECTION Five  Gastrointestinal System
Urinalysis
Urinalysis demonstrates pyuria, hematuria, and/or bacteria in up
to 48% of patients with appendicitis. These abnormalities are due
to the inflamed appendix abutting the ureter, with resultant ure-
teral inflammation. Nevertheless, findings on urinalysis of more
than 30 red blood cells (RBCs)/high-power field or more than 20
WBCs/high-power field are more consistent with urinary tract
infections than appendicitis.
Other Laboratory Tests
A serum or urine pregnancy test is recommended for any female
of childbearing age with abdominal pain. A basic metabolic panel,
liver function tests, and lipase level should be obtained for patients
with suspected appendicitis to assess for electrolyte derangements
and alternative causes of abdominal pain. Procalcitonin does not
currently play a role in the diagnosis of appendicitis due to its
poor predictive value.8
The polymorphonuclear count, in isola-
tion, has no clinical value in the assessment of appendicitis.
Imaging Tests
General Principles
The decision to pursue imaging is based on the provider’s clinical
assessment, which combines the patient’s history, examination,
and laboratory data to decide on the likelihood of appendicitis.
If the likelihood is low (and other significant disease processes
have been excluded), the patient may be discharged from the
emergency department (ED) or observed with serial examinations
in an observation unit (see Chapter e6) or inpatient setting.
However, if there is a concern for appendicitis, imaging should be
carried out.
Currently, patients rarely undergo surgical removal of the
appendix based on clinical features alone. The negative appendec-
tomy rate—the number of normal appendices that are surgically
removed—is far lower when imaging is used.11
Nevertheless, in
rare cases of young men with a classic presentation, the decision
to perform an appendectomy in the absence of imaging may be
pursued at the surgeon’s discretion.
Radiography
Due to their poor sensitivity and specificity, routine radiographs
are of no clinical value in the evaluation of appendicitis. The only
value of radiographs is to assess for other causes of the patient’s
symptoms, such as bowel obstruction or bowel perforation.
However, an ileus mimicking bowel obstruction may occur in
appendicitis due to peritoneal inflammation, and advanced
appendicitis may perforate, resulting in intraperitoneal air on
TABLE 83.2 
Common Maneuvers and Physical Findings Associated With Appendicitis and Their Predictive Valuesa
MANEUVER DESCRIPTION SENSITIVITY AND SPECIFICITY (%)
Iliopsoas (psoas) sign Increased abdominal pain with patient lying on left side while provider passively
extends the patient’s right leg at the hip with both knees extended
Sensitivity: 13–42
Specificity: 79–95
Rovsing’s sign Abdominal pain in the RLQ while palpating the left lower quadrant Sensitivity: 7–68
Specificity: 58–96
Obturator sign Increased abdominal pain in the supine patient as the provider internally and
externally rotates the right leg as it is flexed at the hip
Sensitivity: 8
Specificity: 94
a
Overall poor sensitivity decreases the value of these findings. However, if found, these signs moderately increase the likelihood of having appendicitis.
RLQ, Right lower quadrant.
TABLE 83.3 
Differential Diagnosis in Appendicitis
ALL PATIENTS
FEMALE
PATIENTS
PEDIATRIC
PATIENTS
Nonspecific abdominal pain
Gastroenteritis
Epiploic appendigitis
Ureterolithiasis, nephrolithiasis
Inflammatory bowel disease
Ileus or bowel obstruction
Intestinal perforation
Testicular torsion (males)
Ectopic pregnancy
Ovarian torsion
Pelvic inflammatory
disease
Ovarian cyst
Henoch-Schönlein
purpura
Mesenteric
lymphadenitis
Meckel’s
diverticulum
increasingly high cutoff values for the WBC count (eg, >15,000 or
20,000/mm3
), it is still not significant enough to be used in clinical
practice to diagnose or exclude acute appendicitis.7,8
C-Reactive Protein
The C-reactive protein (CRP) level is a nonspecific, systemic
inflammatory marker synthesized by the liver. It has a poor pre-
dictive value in diagnosing or excluding acute appendicitis.8
An
elevated CRP (>8–10 mg/L) has a sensitivity of 65% to 85%,
specificity of 32% to 87%, positive LR of 1.59 to 4.2, and negative
LR of 0.11 to 3.2.8,9
Some studies have suggested that CRP may be
useful for predicting the severity of appendicitis and likelihood of
complications; however, its value in diagnosing appendicitis lies
in combining CRP with the WBC (see below).10
Combined Inflammatory Markers
The American College of Emergency Physicians clinical policy on
patients with suspected appendicitis states that the combination
of a WBC more than 10,000/mm3
and CRP more than 8 mg/L has
a positive likelihood ratio of 23 and a negative likelihood ratio of
0.03.These combined laboratory findings offer the greatest impact
when excluding appendicitis in patients with a low pretest prob-
ability of the disease. Although more research is needed, based on
the best available evidence, we recommend using the combination
of low WBC (<10,000/mm3
) and CRP (<8 mg/L) in patients
deemed to be low risk for appendicitis, based on the provider’s
clinical assessment, to exclude appendicitis. Alternatively, we do
not believe that there are sufficient data to establish the diagnosis
of appendicitis conclusively based on the combination of elevated
WBC and CRP levels; rather, these should only assist in the deci-
sion making process regarding the need for imaging.
Descargado para Francisco Medina Anderson (fz.medina@hotmail.com) en Organización de Estados Iberoamericanos - Remote Access de ClinicalKey.es por Elsevier en julio 07, 2017.
Para uso personal exclusivamente. No se permiten otros usos sin autorización. Copyright ©2017. Elsevier Inc. Todos los derechos reservados.
1125CHAPTER 83  Acute Appendicitis
pelvic US is an important study to help determine ovarian pathol-
ogy or tuboovarian abscesses. This should be performed before
CT imaging in an attempt to elucidate an alternative diagnosis
and may be completed simultaneously with a graded compression
US to assess for appendicitis.
Computed Tomography
CT of the abdomen and pelvis is considered the test of choice for
definitive assessment of possible appendicitis in nonpregnant
patients. It demonstrates an overall sensitivity of 94% to 100%
and specificity of 91% to 99%, with a positive LR of 9.29 to 13.3,
negative LR of 0.1 to 0.09, and positive predictive value of 95% to
97%.13
CT is accurate and consistent in diagnosing appendicitis
and decreases the negative appendectomy rate. CT is readily avail-
able in most hospitals, can be performed in a rapid fashion, is not
operator-dependent, can be interpreted by most radiologists and
surgeons, and has a greater likelihood of finding an alternative
diagnosis (vs. US; Figs. 83.5 and 83.6).
abdominal radiographs. Therefore, care must be taken to make
the final diagnosis based on radiographic findings, although intra-
peritoneal air often expedites the patient’s disposition to the oper-
ating room.
Graded-Compression Ultrasound
Within the medical community, there is a growing awareness of
the risks associated with ionizing radiation, and efforts are being
made to use methods of diagnosis that reduce or eliminate these
risks.12
Graded compression ultrasound (US) is an imaging tool
commonly used in evaluating patients for appendicitis. It is a
diagnostic technique in which steady pressure is applied with the
US probe to the abdomen to reduce bowel gas and collapse normal
bowel to promote visualization of the appendix. Studies involving
graded compression US for the diagnosis of appendicitis have
reported sensitivities of 75% to 90%, specificities of 83% to 95%,
positive LRs of 4.5 to 5.8, and negative LRs of 0.19 to 0.27, with
an average positive predictive value of 90%.13
Table 83.4 lists US
criteria for the diagnosis of appendicitis.
The benefits in using US for the diagnosis of appendicitis
include decreased cost relative to other imaging modalities, lack
of ionizing radiation exposure, and decreased time to diagnosis.
Limitations of US use include decreased specificity and increased
pain due to the transducer pressure needed for the graded com-
pression process. Most importantly, a number of US examinations
cannot visualize the appendix (ie, nondiagnostic) for a number of
reasons, including lack of operator experience, patient factors (eg,
obesity), superimposed bowel gas, or atypically located appen-
dix.14
In cases with nondiagnostic US findings, the patient typi-
cally requires further imaging with CT (or magnetic resonance
imaging [MRI] in pregnancy) or admission for observation and
serial examinations. Ultrasound is most useful in children, for
whom the risks of ionizing radiation are greatest, and rates of
overweight and obese individuals are lower than adults and preg-
nant females (Figs. 83.3 and 83.4).
A distinction must be made between radiology-based US and
bedside (point of care) US examination performed by an emer-
gency clinician. Recent studies have demonstrated that bedside US
is not as effective at diagnosing appendicitis, with a sensitivity for
diagnosis of 60% to 70%, with specificities of 94% to 98%.15,16
Finally, in women with CMT, masses found on pelvic examina-
tion, or concern for a gynecologic cause of the patient’s symptoms,
TABLE 83.4 
Diagnostic Criteria for Appendicitis on Imaging
ULTRASOUND COMPUTED TOPOGRAPHY MAGNETIC RESONANCE IMAGING
The first two criteria below must be fulfilled: Not all criteria listed below need to be fulfilled but
the combination and severity of these findings
contribute to a diagnosis:
Not all criteria listed need to be fulfilled
but the combination and severity of
these findings contribute to a diagnosis:
•	 Appendiceal diameter > 6–7 mma
Appendiceal diameter (>6 mm with surrounding
inflammation or >8 mm without such changes)
Appendiceal diameter > 7 mm
•	 Noncompressible appendix Appendiceal circumferential wall thickening >2 mm
with mural enhancement (sign of inflammation)
Appendiceal circumferential wall thickening
> 2 mm
Fat stranding (hyperechoic signals associated
with periappendiceal inflammation)
(secondary finding) and peritoneal fluid
Calcified appendicolith Signs of inflammation adjacent to the
appendix, such as fat stranding or
phlegmon formation
Peritoneal fluid surrounding the appendix
(secondary finding)
Signs of periappendiceal inflammation (eg, fat
stranding, clouding of the adjacent mesentery)
Presence of an abscess or a fluid filled
appendix
a
It is important to note that the diameter of a normal nondiseased appendix may be up to 11 mm, so the other findings of appendicitis must be factored in when making the
diagnosis of appendicitis on CT or MRI. Due to the graded compression technique used in ultrasound, there is more certainty regarding diagnostic criteria for appendiceal
diameter.
Fig. 83.3.  Ultrasound image of appendicitis in an 8-year-girl. Note the
dilated noncompressible appendix (thin arrows) and the presence of a
fecalith, with posterior acoustic shadowing (thick arrow). (Courtesy Dr.
Michael Cole, with permission.)
Descargado para Francisco Medina Anderson (fz.medina@hotmail.com) en Organización de Estados Iberoamericanos - Remote Access de ClinicalKey.es por Elsevier en julio 07, 2017.
Para uso personal exclusivamente. No se permiten otros usos sin autorización. Copyright ©2017. Elsevier Inc. Todos los derechos reservados.
1126 PART III  Medicine and Surgery  |  SECTION Five  Gastrointestinal System
To this end, there have been recent studies of low-dose CT
protocols for the diagnosis of appendicitis. These low-dose pro-
tocols decrease the average dose to approximately 2 mSv, with no
detriment in the negative appendectomy rate. However, there is
less diagnostic certainty by radiologists about the diagnosis of
appendicitis with these studies. These are relatively new protocols
that show promise but require more studies before they can be
universally adopted.17,18
Table 83.4 lists CT findings diagnostic of appendicitis. In some
cases, the appendix cannot be visualized. In these cases, if CT
demonstrates no findings of inflammation in the RLQ, it has been
found that appendicitis is unlikely. However, patients with low
amounts of intra-abdominal body fat may not display secondary
signs of inflammation; consequently, these patients may lack this
important marker of appendicitis on CT imaging, leading to false-
negative study results. The term tip appendicitis refers to obstruc-
tion and inflammation limited to the distal tip of the appendix
and is a subtle finding on CT that is a common cause of false-
negative interpretation.19
To assess for appendicitis, CT should be performed with IV
contrast only. Enteric contrast of any type, oral or rectal, contrib-
utes little to the assessment of appendicitis. In addition, studies
have demonstrated that non–contrast-enhanced CT has accept-
able accuracy in diagnosing appendicitis. Furthermore, according
to the American College of Radiology’s appropriateness criteria
for imaging suspected appendicitis, CT imaging with or without
IV contrast are acceptable imaging modalities, with the use of
enteric contrast being deferred to institutional preference. There-
fore, if there are contraindications to IV contrast, there should be
little hesitation to move forward with non–contrast-enhanced CT
for the evaluation of appendicitis.17
Magnetic Resonance Imaging
When considering the evaluation for appendicitis, current evi-
dence supports the use of MRI for assessment in pregnant females
if US is nondiagnostic. MRI has the advantage of not using ion-
izing radiation and is not operator-dependent. However, its use is
limited by its increased cost, increased time required to acquire
images,limited availability,and need for the radiologist or surgeon
The greatest disadvantage of CT is the ionizing radiation. A CT
scan of the abdomen exposes the patient to an average dose of
ionizing radiation equivalent to 8 examination 10 mSv. To put this
in perspective, the average ionizing radiation dose associated with
an abdominal x-ray is 0.7 mSv, and the average dose associated
with coronary angioplasty is 15 mSv. An abdominal CT carries an
excess risk of fatal cancer of 1 in 2000, a value that is even greater
in children. However, this value must be tempered by the fact that
the general population has a lifetime risk of being diagnosed with
cancer of 1 in 3. The risk of radiation increases conversely with
age, with children and fetuses having the greatest risk of adverse
outcomes of radiation due to their smaller body habitus, more
rapidly developing cells, and increased incubation time for genetic
mutations to manifest.
Fig. 83.5.  CT scan with typical findings of acute appendicitis. thick
arrow, thin arrows, dashed line arrow.
Fig. 83.6.  Oral contrast CT scan showing discrete abscess from appen-
diceal perforation, with periappendiceal fat streaking. (Courtesy Jefferson
Radiology, Avon, CT.)
Fat stranding
Discrete abscess
from perforated
appendix
Fig. 83.4.  Graded compression ultrasound scan demonstrating a dilated
noncompressible appendix (thin arrow) representing appendicitis.
Descargado para Francisco Medina Anderson (fz.medina@hotmail.com) en Organización de Estados Iberoamericanos - Remote Access de ClinicalKey.es por Elsevier en julio 07, 2017.
Para uso personal exclusivamente. No se permiten otros usos sin autorización. Copyright ©2017. Elsevier Inc. Todos los derechos reservados.
1127CHAPTER 83  Acute Appendicitis
IV fluids, antipyretics, and antibiotics (see below). The patient’s
pain and nausea should be treated with parenteral opiate analgesia
and antiemetics, respectively. There have been a number of good-
quality studies that support the concept that opiate analgesia does
not negatively affect a patient’s abdominal examination when the
patient has an abdominal condition that requires surgery.26,27
Therefore, parenteral opiate analgesia should not be withheld
from the patient unless there are contraindications to its use (eg,
severe hypotension, allergies). In rare cases, acute appendicitis can
cause severe sepsis or septic shock.
Antibiotic Therapy
Antibiotic therapy should be promptly administered on making
the diagnosis of appendicitis or in patients with suspected appen-
dicitis and severe sepsis or septic shock. The choice of antibiotics
should include broad-spectrum gram-negative and anaerobic
coverage. For nonperforated appendicitis, we recommend cipro-
floxacin, 400 mg IV, and metronidazole (Flagyl), 500 mg IV; or
ceftriaxone, 1g IV, and metronidazole, 500 mg IV; or ampicillin-
sulbactam, 3g IV monotherapy. For perforated appendicitis, we
recommend broader spectrum antibiotics, such as piperacillin-
tazobactam, 3.375 to 4.5g IV, cefepime, 2 g IV, or imipenem-
cilastatin, 500 mg IV. Methicillin-resistant Staphylococcus aureus
(MRSA) coverage is not typically needed to treat appendicitis but
may be considered if the patient has previously known MRSA
colonization.28
Definitive Treatment
Definitive treatment of acute appendicitis will depend on whether
there are associated complications, and all decisions should be
made in consultation with the surgical service. Nonperforated
appendicitis with a well-circumscribed abscess should be treated
with IV antibiotics and percutaneous drainage. Perforated appen-
dicitis with or without abscess is treated with IV antibiotics and
urgent operative intervention.28
Nonperforated appendicitis without abscess (ie,uncomplicated
appendicitis) is traditionally treated with IV antibiotics and surgi-
cal removal of the inflamed appendix. However, recent and his-
torical data have demonstrated that conservative treatment of
appendicitis with antibiotic therapy and a period of inpatient
observation may be a viable treatment option for certain patients.
There is historical precedence for nonoperative management of
appendicitis,and recent studies have found that there may be value
in risk-stratifying patients with appendicitis based on their CT
findings. In appendicitis with low-risk features, antibiotic therapy
with a period of inpatient observation is a feasible option.29,30
Features associated with failed conservative management include
the presence of a fecalith, abscess, tumor, or fluid collection or
appendiceal diameter of more than 1.1 cm.31
In patients with any
of these features, operative intervention is preferred.
A minority of patients treated conservatively may fail the inpa-
tient observation period and still require surgery; a minority of
those discharged after conservative treatment carry the risk of
recurrence of appendicitis. However, with a negative appendec-
tomy rate of 3.6% to 10% and a complication rate as high as
18%—including small bowel obstruction, adhesions, surgical site
infection, and abscess formation—nonoperative care is an option
worth considering.32
The decision regarding definitive treatment
of acute appendicitis should be made in consultation with the
surgical service and the risks and benefits of conservative treat-
ment versus surgical intervention should be frankly discussed
with the patient, surgeon, and emergency clinician.
When the decision is made to proceed with surgical removal
of the appendix, in uncomplicated appendicitis, delaying surgery
up to 12 hours after diagnosis is made (eg, “waiting until the
to be skilled in MRI scan interpretation. MRI demonstrates a
sensitivity of 85% to 100%, specificity of 95% to 99.2, average
positive predictive value of 92.4, and average negative predictive
value of 99.7.20
Table 83.4 lists MRI criteria for the diagnosis of
appendicitis.
In pregnant patients, IV gadolinium contrast should not be
used when evaluating for appendicitis due to potentially harmful
effects on the fetus.21
Enteric contrast may be used at the discre-
tion of the interpreting radiologist or per institutional protocol.
Combined Imaging Pathways
An imaging pathways that combine US and CT, in which abdomi-
nopelvic CT is performed if the graded compression US is non-
diagnostic or negative, have demonstrated combined sensitivities
of 94% to 99%, specificities of 91% to 97.5%, and significant
reductions in CT utilization.22,23
It has been projected that this
pathway would save $547/patient in imaging costs and $25
million/year in aggregate by reducing imaging costs, unnecessary
surgeries, and unnecessary hospitalizations, not to mention
decreased radiation exposure.24
As institutions increase their
experience with the use of US to diagnose appendicitis, we think
that a combined US-CT pathway will gain acceptance and improve
health care delivery.
Interestingly, a so-called radiation-free imaging pathway that
combines US and MRI, in which abdominopelvic MRI is per-
formed if the US is nondiagnostic or negative, has been recently
studied in the emergency pediatric population, with outcomes
similar to those of the combined US-CT pathway. However, at this
time, there is a paucity of sufficient data and lack of institutional
resources to suggest the routine use of this approach.25
Summary of Imaging Methods
Fig. 83.2 illustrates a suggested pathway regarding imaging. For
nonpregnant patients, graded compression US may be first con-
sidered. In nonpregnant females, a pelvic US may also be consid-
ered to assess for pelvic pathology. The ability to visualize the
appendix on US is institution-dependent, and the provider’s deci-
sion to use US initially may depend on the institution’s level of
experience with this modality. If the US studies are negative or
nondiagnostic (ie, no appendix is visualized and no alternative
pathology is noted), the patient may undergo CT imaging of the
abdomen and pelvis with IV contrast (no PO contrast). An alter-
native to CT imaging in low-risk cases with nondiagnostic US is
admission for observation and serial examinations.
If the patient is pregnant, graded compression and pelvic US
should always be the initial studies of choice, followed by MRI of
the abdomen without IV contrast in cases of nondiagnostic or
negative US findings. If MRI is not available, and transfer to a
facility with MRI capabilities is not feasible, then, after consulta-
tion with a radiologist, general surgeon, and obstetrician, abdomi-
nal CT scanning with IV contrast may be considered. However, in
low-risk cases, admission for observation and serial examinations
is an acceptable alternative.
MANAGEMENT
Supportive Care
Decisions surrounding supportive care will depend on the patient’s
condition and needs. Supportive care should be initiated prior to
a definitive diagnosis and should continue until the patient leaves
the ED. Patients should remain NPO. IV fluids (normal saline or
lactated Ringer’s) may be administered to maintain hydration and
support hypotensive patients. Systemic signs of infection are more
common in perforated appendicitis and should be supported by
Descargado para Francisco Medina Anderson (fz.medina@hotmail.com) en Organización de Estados Iberoamericanos - Remote Access de ClinicalKey.es por Elsevier en julio 07, 2017.
Para uso personal exclusivamente. No se permiten otros usos sin autorización. Copyright ©2017. Elsevier Inc. Todos los derechos reservados.
1128 PART III  Medicine and Surgery  |  SECTION Five  Gastrointestinal System
based on imaging or, rarely, clinical assessment alone. In this case,
antibiotics should be initiated, surgical consultation should be
obtained, and the patient should be admitted for operative inter-
vention or, in select cases, IV antibiotics and observation. Based
on clinical and laboratory assessment, the risk of appendicitis is
low, and no imaging study was performed. In this case, the patient
may be discharged home if he or she is reliable, has improved
clinical status (ie, feels better), and understands the provider’s
thought process and precautionary instructions. Alternatively, if
these criteria are not met, the patient may be transferred to an
observation unit or hospitalized for serial examinations. If the
patient’s imaging results are inconclusive, or if they are negative
but the patient is still symptomatic, the patient may be admitted
for observation, symptomatic treatment, serial examinations, and
kept NPO, although select patients in this category may still be
discharged at the provider’s discretion.
morning”) is acceptable and does not lead to worse clinical out-
comes; one recent study has demonstrated that inpatient delay of
appendectomy by up to 24 hours does not result in worse out-
comes.33
Although limited in-hospital delay of operative care has
not been shown to increase perforation risk or morbidity, delay
by patients initially seeking care does increase the risk of perfora-
tion and associated morbidity. Lack of insurance, male gender,
and a greater number of comorbid conditions are factors associ-
ated with increased perforation risk. The choice of laparoscopic
versus open appendectomy is made by the surgeon; however, lapa-
roscopic appendectomy has become the current method of choice.
DISPOSITION
There are three possible disposition pathways when a diagnosis of
appendicitis is considered. A diagnosis of appendicitis is made
•	 Appendicitis is a progressive illness caused by appendiceal luminal
distention followed by appendiceal wall ischemia, transmural
inflammation, and eventual perforation, with resultant peritonitis.
•	 Clinical history, physical examination, and laboratory findings need to
be combined to formulate a comprehensive assessment. No one
finding can definitively diagnose or exclude appendicitis.
•	 The most useful historical features in evaluating appendicitis are RLQ
pain, pain preceding vomiting, and migration of pain to the RLQ.
•	 The most useful physical findings in evaluating appendicitis are RLQ
tenderness and rigidity.
•	 Cervical motion tenderness is not specific for pelvic pathology and is
found in up to 28% of females with appendicitis.
•	 A rectal examination contributes little and should not be routinely
performed in the evaluation of appendicitis.
•	 The white blood cell count alone is neither sensitive nor specific for
appendicitis and offers little in the evaluation of appendicitis.
•	 When clinicians have a low pretest possibility for appendicitis, the
combination of a WBC count below 10,000/mm3
and CRP level
below 8 mg/L support the exclusion of appendicitis as a likely
diagnosis.
•	 Nonoperative management of acute appendicitis (IV antibiotics,
admission) is gaining support. The patient should not have high-risk
features (eg, presence of a fecalith, abscess, tumor, or fluid collection
or appendiceal diameter >1.1 cm) and should be made aware of the
risk of failed observation as an inpatient or recurrent appendicitis
once discharged, both of which would then require surgical removal
of the appendix.
•	 Once the diagnosis of appendicitis is made, in-hospital delay of
appendectomy of up to 12 hours has not demonstrated negative
outcomes when compared to emergent operative care.
KEY CONCEPTS
The references for this chapter can be found online by accessing the accompanying Expert Consult website.
Descargado para Francisco Medina Anderson (fz.medina@hotmail.com) en Organización de Estados Iberoamericanos - Remote Access de ClinicalKey.es por Elsevier en julio 07, 2017.
Para uso personal exclusivamente. No se permiten otros usos sin autorización. Copyright ©2017. Elsevier Inc. Todos los derechos reservados.
1128.e1CHAPTER 83  Acute Appendicitis
REFERENCES
1.	 Smith HF, Parker W, Kotzé SH, et al: Multiple independent appearances of the cecal
appendix in mammalian evolution and an investigation of related ecological and
anatomical factors. Comptes Rendus Palevol 12:339–354, 2013.
2.	 Im GY, Modayil RJ, Lin CT, et al: The appendix may protect against Clostridium
difficile recurrence. Clin Gastroenterol Hepatol 9:1072–1077, 2011.
3.	 Clanton J, Subichin M, Drolshagen K, et al: Fulminant Clostridium difficile infection:
an association with prior appendectomy? World J Gastrointest Surg 5:233–238, 2013.
4.	 Hendahewa R, Shekhar A, Ratnayake S: The dilemma of stump appendicitis—a case
report and literature review. Int J Surg Case Rep 14:101–103, 2015.
4a.  Laméris W, van Randen A, Go PM, et al: Single and combined diagnostic value of
clinical features and laboratory tests in acute appendicitis. Acad Emerg Med 16(9):
835–842, 2009.
5.	 Brown TW, McCarthy ML, Kelen GD, et al: An epidemiologic study of closed emer-
gency department malpractice claims in a national database of physician malpractice
insurers. Acad Emerg Med 17:553–560, 2010.
6.	 Takada T, Nishiwaki H, Yamamoto Y, et al: The role of digital rectal examination for
diagnosis of acute appendicitis: a systematic review and meta-analysis. PLoS ONE
10:e0136996, 2015.
7.	 Howell JM, Eddy OL, Lukens TW, et al: Clinical policy: critical issues in the evaluation
and management of emergency department patients with suspected appendicitis.
Ann Emerg Med 55:71–116, 2010.
8.	 Yu C-W, Juan L-I, Wu M-H, et al: Systematic review and meta-analysis of the diag-
nostic accuracy of procalcitonin, C-reactive protein and white blood cell count for
suspected acute appendicitis. Br J Surg 100:322–329, 2013.
9.	 Farooqui W, Pommergaard H-C, Burcharth J, et al: The diagnostic value of a panel
of serological markers in acute appendicitis. Scand J Surg 104:72–78, 2015.
10.	 Shindoh J, Niwa H, Kawai K, et al: Diagnostic power of inflammatory markers in
predicting severity of appendicitis. Hepatogastroenterology 58:2003–2006, 2011.
11.	 Bachur RG, Hennelly K, Callahan MJ, et al: Diagnostic imaging and negative appen-
dectomy rates in children: effects of age and gender. Pediatrics 129:877–884, 2012.
12.	 Miglioretti DL, Johnson E, Williams A, et al: The use of computed tomography in
pediatrics and the associated radiation exposure and estimated cancer risk. JAMA
Pediatr 167:700–707, 2013.
13.	 van Randen A, Laméris W, van Es HW, et al: A comparison of the accuracy of ultra-
sound and computed tomography in common diagnoses causing acute abdominal
pain. Eur Radiol 21:1535–1545, 2011.
14.	 Abo A, Shannon M, Taylor G, et al: The influence of body mass index on the accuracy
of ultrasound and computed tomography in diagnosing appendicitis in children.
Pediatr Emerg Care 27:731–736, 2011.
15.	 Mallin M, Craven P, Ockerse P, et al: Diagnosis of appendicitis by bedside ultrasound
in the ED. Am J Emerg Med 33:430–432, 2015.
16.	 Elikashvili I, Tay ET, Tsung JW: The effect of point-of-care ultrasonography on
emergency department length of stay and computed tomography utilization in chil-
dren with suspected appendicitis. Acad Emerg Med 21:163–170, 2014.
17.	 Smith MP, Katz DS, Lalani T, et al: ACR Appropriateness Criteria® right lower quad-
rant pain—suspected appendicitis. Ultrasound Q 31:85–91, 2015.
18.	 Kim K, Kim YH, Kim SY, et al: Low-dose abdominal CT for evaluating suspected
appendicitis. N Engl J Med 366:1596–1605, 2012.
19.	 Gaetke-Udager K, Maturen KE, Hammer SG: Beyond acute appendicitis: imaging and
pathologic spectrum of appendiceal pathology. Emerg Radiol 21:535–542, 2014.
20.	 Burke LMB, Bashir MR, Miller FH, et al: Magnetic resonance imaging of acute
appendicitis in pregnancy: a 5-year multiinstitutional study. Am J Obstet Gynecol
213:693.e1–693.e6, 2015.
21.	 Expert Panel on MR Safety, Kanal E, Barkovich AJ, et al: ACR guidance document on
MR safe practices: 2013. J Magn Reson Imaging 37:501–530, 2013.
22.	 Thirumoorthi AS, Fefferman NR, Ginsburg HB, et al: Managing radiation exposure
in children—reexamining the role of ultrasound in the diagnosis of appendicitis.
J Pediatr Surg 47:2268–2272, 2012.
23.	 Polites SF, Mohamed MI, Habermann EB, et al: A simple algorithm reduces com-
puted tomography use in the diagnosis of appendicitis in children. Surgery 156:448–
454, 2014.
24.	 Bachur RG, Levy JA, Callahan MJ, et al: Effect of reduction in the use of computed
tomography on clinical outcomes of appendicitis. JAMA Pediatr 169:755–760, 2015.
25.	 Aspelund G, Fingeret A, Gross E, et al: Ultrasonography/MRI versus CT for diagnos-
ing appendicitis. Pediatrics 133:586–593, 2014.
26.	 Manterola C,Vial M, Moraga J, et al: Analgesia in patients with acute abdominal pain.
Cochrane Database Syst Rev (1):CD005660, 2011.
27.	 Poonai N, Paskar D, Konrad S-L, et al: Opioid analgesia for acute abdominal pain in
children: A systematic review and meta-analysis. Acad Emerg Med 21:1183–1192,
2014.
28.	 Solomkin JS, Mazuski JE, Bradley JS, et al: Diagnosis and management of compli-
cated intra-abdominal infection in adults and children: guidelines by the Surgical
Infection Society and the Infectious Diseases Society of America. Clin Infect Dis
50:133–164, 2010.
29.	 Salminen P, Paajanen H, Rautio T, et al: Antibiotic therapy vs appendectomy for
treatment of uncomplicated acute appendicitis: the APPAC randomized clinical trial.
JAMA 313:2340–2348, 2015.
30.	 Vons C, Barry C, Maitre S, et al: Amoxicillin plus clavulanic acid versus appendicec-
tomy for treatment of acute uncomplicated appendicitis: an open-label, non-
inferiority, randomised controlled trial. Lancet 377:1573–1579, 2011.
31.	 Shindoh J, Niwa H, Kawai K, et al: Predictive factors for negative outcomes in initial
non-operative management of suspected appendicitis. J Gastrointest Surg 14:309–
314, 2010.
32.	 Seetahal SA, Bolorunduro OB, Sookdeo TC, et al: Negative appendectomy: a 10-year
review of a nationally representative sample. Am J Surg 201:433–437, 2011.
33.	 Drake FT, Mottey NE, Farrokhi ET, et al: Time to appendectomy and risk of perfora-
tion in acute appendicitis. JAMA Surg 149:837–844, 2014.
34.	 Ashdown HF, D’Souza N, Karim D, et al: Pain over speed bumps in diagnosis of acute
appendicitis: diagnostic accuracy study. BMJ 345:e8012, 2012.
35.	 Laurell H, Hansson L-E, Gunnarsson U: Manifestations of acute appendicitis: a
prospective study on acute abdominal pain. Dig Surg 30:198–206, 2013.
CHAPTER 83: QUESTIONS & ANSWERS
83.1.	 What percentage of women with acute appendicitis have
accompanying cervical motion tenderness (CMT)?
A.	10%
B.	 15%
C.	20%
D.	25%
E.	 30%
Answer: D. Prior to the advent of routine imaging of the appen-
dix, as many as 25% of women with acute appendicitis were ini-
tially misdiagnosed because of the presence of CMT.
83.2.	 Which of the following statements regarding
ultrasonographic visualization of the appendix is true?
A.	A compressible appendix is a positive finding.
B.	 An appendiceal diameter greater than 6 or 7 mm is a
positive finding.
C.	The sensitivity of ultrasound for appendicitis is 94% to
98%.
D.	Ultrasonography has good reliability for detecting a
retrocecal appendix.
E.	 Ultrasonography compares favorably with computed
tomography (CT) scanning for the detection of
appendicitis.
Answer: B. A noncompressible appendix with a diameter greater
than 6 or 7 mm in a setting of clinical appendicitis is considered
a positive finding. Ultrasound sensitivities are 75% to 90%. It is a
less useful modality in the obese, those with peritoneal adhesions,
and those with a retrocecal appendix. The sensitivity of helical CT
scanning with rectal contrast approaches 98%, much higher than
ultrasonography.
83.3.	 A 27-year-old G3P2 woman at 22 weeks of gestation
presents with 2 days of right lower quadrant (RLQ)
abdominal pain. It began midline and later became more
pronounced in the RLQ. The physical examination was
remarkable for RLQ tenderness without rebound. The
gynecologic examination was negative except for a
nontender gravid uterus, with good fetal movement by
transabdominal ultrasound. Urinalysis showed 8 to 10
white blood cells (WBCs)/high-power field (HPF) and
occasional bacteria. Complete blood count (CBC) showed
a WBC count of 12,700/mm3
with 77% neutrophils.
Hemoglobin level was 11 g/dL. RLQ ultrasound was
limited, with no visualization of a normal or abnormal
appendix, and transvaginal ultrasound did not show an
obvious gynecologic or obstetric problem. Repeat
examination showed continued RLQ tenderness. What is
the most appropriate intervention?
A.	Administer cephalexin for urinary tract infection and
schedule a 48-hour clinic recheck
B.	 Admit for observation and serial examination
Descargado para Francisco Medina Anderson (fz.medina@hotmail.com) en Organización de Estados Iberoamericanos - Remote Access de ClinicalKey.es por Elsevier en julio 07, 2017.
Para uso personal exclusivamente. No se permiten otros usos sin autorización. Copyright ©2017. Elsevier Inc. Todos los derechos reservados.
1128.e2 PART III  Medicine and Surgery  |  SECTION Five  Gastrointestinal System
C.	Obtain surgical consultation for laparotomy
D.	Order a CT scan of the abdomen.
E.	 Order a magnetic resonance imaging (MRI) scan
Answer: E. MRI scanning for appendicitis may be helpful in preg-
nant women, in whom the avoidance of radiation exposure is a
significant consideration, and exploratory surgery carries addi-
tional risks.
83.4.	 In men and children with classic symptoms and signs of
appendicitis, what is the most appropriate initial
intervention?
A.	Antibiotics and serial abdominal examinations
B.	 CT scan of the abdomen
C.	MRI scan of the abdomen
D.	Surgery
E.	 Ultrasonography
Answer: E. In men and children with classic appendicitis, imaging
adds little to the evaluation and only exposes patients to unneces-
sary radiation. However, it has become less and less common for
a patient with a history and examination concerning for appen-
dicitis to undergo surgery without further imaging. Ultrasound is
the most appropriate initial intervention because it uses no radia-
tion and can often visualize and diagnose appendicitis without
significant delay. Graded compression ultrasound for appendicitis
is specific but lacks the sensitivity of CT scan so, if the appendix
is not visualized, a discussion can be had with the general surgeon
to determine if it is necessary to obtain further information (via
CT or MRI).
Descargado para Francisco Medina Anderson (fz.medina@hotmail.com) en Organización de Estados Iberoamericanos - Remote Access de ClinicalKey.es por Elsevier en julio 07, 2017.
Para uso personal exclusivamente. No se permiten otros usos sin autorización. Copyright ©2017. Elsevier Inc. Todos los derechos reservados.

More Related Content

What's hot

Umbilical hernia by Dr. kiran maindale
Umbilical hernia by Dr. kiran maindaleUmbilical hernia by Dr. kiran maindale
Umbilical hernia by Dr. kiran maindalekiran Maindale
 
Intestinal obstruction
Intestinal obstructionIntestinal obstruction
Intestinal obstructionsyed ubaid
 
Intestinal obstruction, BOWEL OBSTRUCTION
Intestinal obstruction, BOWEL OBSTRUCTIONIntestinal obstruction, BOWEL OBSTRUCTION
Intestinal obstruction, BOWEL OBSTRUCTIONpankaj rana
 
Anorectal malformation
Anorectal malformationAnorectal malformation
Anorectal malformationArifa T N
 
Gastro intestinal perforation
Gastro intestinal perforationGastro intestinal perforation
Gastro intestinal perforationsonali Harsh Raj
 
Chapter 26 Appendix
Chapter 26 AppendixChapter 26 Appendix
Chapter 26 Appendixhuang.shuo
 
Intussusception (2)
Intussusception (2)Intussusception (2)
Intussusception (2)Rajiv Lal
 
Tuberculosis Abdomen
Tuberculosis AbdomenTuberculosis Abdomen
Tuberculosis AbdomenANILKUMAR BR
 
Acute Calculous Cholecystitis
Acute Calculous CholecystitisAcute Calculous Cholecystitis
Acute Calculous CholecystitisSun Yai-Cheng
 
Cholecystitis cholelithiasis-presentation
Cholecystitis cholelithiasis-presentationCholecystitis cholelithiasis-presentation
Cholecystitis cholelithiasis-presentationAnshu Yadav
 
Neonatal intestinal obstruction
Neonatal intestinal obstructionNeonatal intestinal obstruction
Neonatal intestinal obstructionKhaled Bahaaeldin
 
Acute appendicitis &amp;lump
Acute appendicitis &amp;lumpAcute appendicitis &amp;lump
Acute appendicitis &amp;lumpsyed ubaid
 

What's hot (20)

Umbilical hernia by Dr. kiran maindale
Umbilical hernia by Dr. kiran maindaleUmbilical hernia by Dr. kiran maindale
Umbilical hernia by Dr. kiran maindale
 
Appendicites
AppendicitesAppendicites
Appendicites
 
Appendicitis
AppendicitisAppendicitis
Appendicitis
 
intestinal obstruction
intestinal obstructionintestinal obstruction
intestinal obstruction
 
Intestinal obstruction
Intestinal obstructionIntestinal obstruction
Intestinal obstruction
 
Intestinal obstruction, BOWEL OBSTRUCTION
Intestinal obstruction, BOWEL OBSTRUCTIONIntestinal obstruction, BOWEL OBSTRUCTION
Intestinal obstruction, BOWEL OBSTRUCTION
 
Stump appendicitis
Stump appendicitisStump appendicitis
Stump appendicitis
 
Splenectomy
Splenectomy Splenectomy
Splenectomy
 
Anorectal malformation
Anorectal malformationAnorectal malformation
Anorectal malformation
 
Gastro intestinal perforation
Gastro intestinal perforationGastro intestinal perforation
Gastro intestinal perforation
 
Intestinal obstruction
Intestinal obstructionIntestinal obstruction
Intestinal obstruction
 
Chapter 26 Appendix
Chapter 26 AppendixChapter 26 Appendix
Chapter 26 Appendix
 
Appendicitis
AppendicitisAppendicitis
Appendicitis
 
Anorectal malformation
Anorectal malformationAnorectal malformation
Anorectal malformation
 
Intussusception (2)
Intussusception (2)Intussusception (2)
Intussusception (2)
 
Tuberculosis Abdomen
Tuberculosis AbdomenTuberculosis Abdomen
Tuberculosis Abdomen
 
Acute Calculous Cholecystitis
Acute Calculous CholecystitisAcute Calculous Cholecystitis
Acute Calculous Cholecystitis
 
Cholecystitis cholelithiasis-presentation
Cholecystitis cholelithiasis-presentationCholecystitis cholelithiasis-presentation
Cholecystitis cholelithiasis-presentation
 
Neonatal intestinal obstruction
Neonatal intestinal obstructionNeonatal intestinal obstruction
Neonatal intestinal obstruction
 
Acute appendicitis &amp;lump
Acute appendicitis &amp;lumpAcute appendicitis &amp;lump
Acute appendicitis &amp;lump
 

Similar to Apendicitis 2

Bohomolets Surgery 4th year Lecture #3
Bohomolets Surgery 4th year Lecture #3Bohomolets Surgery 4th year Lecture #3
Bohomolets Surgery 4th year Lecture #3Dr. Rubz
 
Acute appendicitis __
Acute appendicitis __Acute appendicitis __
Acute appendicitis __MpPm4
 
J.1365 2036.2005.02668.x
J.1365 2036.2005.02668.xJ.1365 2036.2005.02668.x
J.1365 2036.2005.02668.xAgung Haryadi
 
Appendix Pp For Online
Appendix Pp For OnlineAppendix Pp For Online
Appendix Pp For Onlinesashehri
 
Special populations with appendicitis
Special populations with appendicitisSpecial populations with appendicitis
Special populations with appendicitisnuaman danawar
 
Abdominal Splenosiscausing Hydronephrosis- A Case Report
Abdominal Splenosiscausing Hydronephrosis- A Case ReportAbdominal Splenosiscausing Hydronephrosis- A Case Report
Abdominal Splenosiscausing Hydronephrosis- A Case Reportsuppubs1pubs1
 
Abdominal Splenosiscausing Hydronephrosis- A Case Report
Abdominal Splenosiscausing Hydronephrosis- A Case ReportAbdominal Splenosiscausing Hydronephrosis- A Case Report
Abdominal Splenosiscausing Hydronephrosis- A Case Reportsemualkaira
 
appendicitis_compress.pdf
appendicitis_compress.pdfappendicitis_compress.pdf
appendicitis_compress.pdfRolakThapa
 
“Little Old Ladies Hernia”: A Case Report and Review of Literature_Crimson Pu...
“Little Old Ladies Hernia”: A Case Report and Review of Literature_Crimson Pu...“Little Old Ladies Hernia”: A Case Report and Review of Literature_Crimson Pu...
“Little Old Ladies Hernia”: A Case Report and Review of Literature_Crimson Pu...CrimsonPublishersAICS
 
Bowel obstruction - Radiology Clinics 2015
Bowel obstruction - Radiology Clinics 2015Bowel obstruction - Radiology Clinics 2015
Bowel obstruction - Radiology Clinics 2015Hamilton Delgado
 
Gastrointestinal Problems In Children
Gastrointestinal Problems In ChildrenGastrointestinal Problems In Children
Gastrointestinal Problems In ChildrenDJ CrissCross
 
Kastenberg 2013 clinics-in-perinatology
Kastenberg 2013 clinics-in-perinatologyKastenberg 2013 clinics-in-perinatology
Kastenberg 2013 clinics-in-perinatologyMOHAMMAD QUAYYUM
 
Seminar Presentation On Appendicitis (1).pptx
Seminar Presentation On Appendicitis (1).pptxSeminar Presentation On Appendicitis (1).pptx
Seminar Presentation On Appendicitis (1).pptxRebiraWorkineh
 
Idiopathic Peritoneal Sclerosis: Case Presentation, And Literature Review
Idiopathic Peritoneal Sclerosis: Case Presentation, And Literature ReviewIdiopathic Peritoneal Sclerosis: Case Presentation, And Literature Review
Idiopathic Peritoneal Sclerosis: Case Presentation, And Literature Reviewsemualkaira
 
Causes and Consequences of Back Pain
Causes and Consequences of Back PainCauses and Consequences of Back Pain
Causes and Consequences of Back Painkomalicarol
 
Neonatal Necrotizing Enterocolitis
Neonatal Necrotizing EnterocolitisNeonatal Necrotizing Enterocolitis
Neonatal Necrotizing Enterocolitissumona keya
 
Neonatal Necrotizing Enterocolitis
Neonatal Necrotizing EnterocolitisNeonatal Necrotizing Enterocolitis
Neonatal Necrotizing Enterocolitissumona keya
 
Acute ppendicitis case
Acute ppendicitis caseAcute ppendicitis case
Acute ppendicitis caseTimothy Zagada
 

Similar to Apendicitis 2 (20)

Bohomolets Surgery 4th year Lecture #3
Bohomolets Surgery 4th year Lecture #3Bohomolets Surgery 4th year Lecture #3
Bohomolets Surgery 4th year Lecture #3
 
Acute appendicitis __
Acute appendicitis __Acute appendicitis __
Acute appendicitis __
 
J.1365 2036.2005.02668.x
J.1365 2036.2005.02668.xJ.1365 2036.2005.02668.x
J.1365 2036.2005.02668.x
 
Appendix Pp For Online
Appendix Pp For OnlineAppendix Pp For Online
Appendix Pp For Online
 
Special populations with appendicitis
Special populations with appendicitisSpecial populations with appendicitis
Special populations with appendicitis
 
Abdominal Splenosiscausing Hydronephrosis- A Case Report
Abdominal Splenosiscausing Hydronephrosis- A Case ReportAbdominal Splenosiscausing Hydronephrosis- A Case Report
Abdominal Splenosiscausing Hydronephrosis- A Case Report
 
Abdominal Splenosiscausing Hydronephrosis- A Case Report
Abdominal Splenosiscausing Hydronephrosis- A Case ReportAbdominal Splenosiscausing Hydronephrosis- A Case Report
Abdominal Splenosiscausing Hydronephrosis- A Case Report
 
appendicitis_compress.pdf
appendicitis_compress.pdfappendicitis_compress.pdf
appendicitis_compress.pdf
 
Appendix
AppendixAppendix
Appendix
 
Appendicitis
AppendicitisAppendicitis
Appendicitis
 
“Little Old Ladies Hernia”: A Case Report and Review of Literature_Crimson Pu...
“Little Old Ladies Hernia”: A Case Report and Review of Literature_Crimson Pu...“Little Old Ladies Hernia”: A Case Report and Review of Literature_Crimson Pu...
“Little Old Ladies Hernia”: A Case Report and Review of Literature_Crimson Pu...
 
Bowel obstruction - Radiology Clinics 2015
Bowel obstruction - Radiology Clinics 2015Bowel obstruction - Radiology Clinics 2015
Bowel obstruction - Radiology Clinics 2015
 
Gastrointestinal Problems In Children
Gastrointestinal Problems In ChildrenGastrointestinal Problems In Children
Gastrointestinal Problems In Children
 
Kastenberg 2013 clinics-in-perinatology
Kastenberg 2013 clinics-in-perinatologyKastenberg 2013 clinics-in-perinatology
Kastenberg 2013 clinics-in-perinatology
 
Seminar Presentation On Appendicitis (1).pptx
Seminar Presentation On Appendicitis (1).pptxSeminar Presentation On Appendicitis (1).pptx
Seminar Presentation On Appendicitis (1).pptx
 
Idiopathic Peritoneal Sclerosis: Case Presentation, And Literature Review
Idiopathic Peritoneal Sclerosis: Case Presentation, And Literature ReviewIdiopathic Peritoneal Sclerosis: Case Presentation, And Literature Review
Idiopathic Peritoneal Sclerosis: Case Presentation, And Literature Review
 
Causes and Consequences of Back Pain
Causes and Consequences of Back PainCauses and Consequences of Back Pain
Causes and Consequences of Back Pain
 
Neonatal Necrotizing Enterocolitis
Neonatal Necrotizing EnterocolitisNeonatal Necrotizing Enterocolitis
Neonatal Necrotizing Enterocolitis
 
Neonatal Necrotizing Enterocolitis
Neonatal Necrotizing EnterocolitisNeonatal Necrotizing Enterocolitis
Neonatal Necrotizing Enterocolitis
 
Acute ppendicitis case
Acute ppendicitis caseAcute ppendicitis case
Acute ppendicitis case
 

Recently uploaded

Call Girls Service in Bommanahalli - 7001305949 with real photos and phone nu...
Call Girls Service in Bommanahalli - 7001305949 with real photos and phone nu...Call Girls Service in Bommanahalli - 7001305949 with real photos and phone nu...
Call Girls Service in Bommanahalli - 7001305949 with real photos and phone nu...narwatsonia7
 
Russian Call Girls Chickpet - 7001305949 Booking and charges genuine rate for...
Russian Call Girls Chickpet - 7001305949 Booking and charges genuine rate for...Russian Call Girls Chickpet - 7001305949 Booking and charges genuine rate for...
Russian Call Girls Chickpet - 7001305949 Booking and charges genuine rate for...narwatsonia7
 
Call Girls Yelahanka Just Call 7001305949 Top Class Call Girl Service Available
Call Girls Yelahanka Just Call 7001305949 Top Class Call Girl Service AvailableCall Girls Yelahanka Just Call 7001305949 Top Class Call Girl Service Available
Call Girls Yelahanka Just Call 7001305949 Top Class Call Girl Service Availablenarwatsonia7
 
Call Girls Colaba Mumbai ❤️ 9920874524 👈 Cash on Delivery
Call Girls Colaba Mumbai ❤️ 9920874524 👈 Cash on DeliveryCall Girls Colaba Mumbai ❤️ 9920874524 👈 Cash on Delivery
Call Girls Colaba Mumbai ❤️ 9920874524 👈 Cash on Deliverynehamumbai
 
Call Girls Service Chennai Jiya 7001305949 Independent Escort Service Chennai
Call Girls Service Chennai Jiya 7001305949 Independent Escort Service ChennaiCall Girls Service Chennai Jiya 7001305949 Independent Escort Service Chennai
Call Girls Service Chennai Jiya 7001305949 Independent Escort Service ChennaiNehru place Escorts
 
Sonagachi Call Girls Services 9907093804 @24x7 High Class Babes Here Call Now
Sonagachi Call Girls Services 9907093804 @24x7 High Class Babes Here Call NowSonagachi Call Girls Services 9907093804 @24x7 High Class Babes Here Call Now
Sonagachi Call Girls Services 9907093804 @24x7 High Class Babes Here Call NowRiya Pathan
 
Call Girls Horamavu WhatsApp Number 7001035870 Meeting With Bangalore Escorts
Call Girls Horamavu WhatsApp Number 7001035870 Meeting With Bangalore EscortsCall Girls Horamavu WhatsApp Number 7001035870 Meeting With Bangalore Escorts
Call Girls Horamavu WhatsApp Number 7001035870 Meeting With Bangalore Escortsvidya singh
 
Hi,Fi Call Girl In Mysore Road - 7001305949 | 24x7 Service Available Near Me
Hi,Fi Call Girl In Mysore Road - 7001305949 | 24x7 Service Available Near MeHi,Fi Call Girl In Mysore Road - 7001305949 | 24x7 Service Available Near Me
Hi,Fi Call Girl In Mysore Road - 7001305949 | 24x7 Service Available Near Menarwatsonia7
 
High Profile Call Girls Jaipur Vani 8445551418 Independent Escort Service Jaipur
High Profile Call Girls Jaipur Vani 8445551418 Independent Escort Service JaipurHigh Profile Call Girls Jaipur Vani 8445551418 Independent Escort Service Jaipur
High Profile Call Girls Jaipur Vani 8445551418 Independent Escort Service Jaipurparulsinha
 
Low Rate Call Girls Pune Esha 9907093804 Short 1500 Night 6000 Best call girl...
Low Rate Call Girls Pune Esha 9907093804 Short 1500 Night 6000 Best call girl...Low Rate Call Girls Pune Esha 9907093804 Short 1500 Night 6000 Best call girl...
Low Rate Call Girls Pune Esha 9907093804 Short 1500 Night 6000 Best call girl...Miss joya
 
Call Girl Coimbatore Prisha☎️ 8250192130 Independent Escort Service Coimbatore
Call Girl Coimbatore Prisha☎️  8250192130 Independent Escort Service CoimbatoreCall Girl Coimbatore Prisha☎️  8250192130 Independent Escort Service Coimbatore
Call Girl Coimbatore Prisha☎️ 8250192130 Independent Escort Service Coimbatorenarwatsonia7
 
Kesar Bagh Call Girl Price 9548273370 , Lucknow Call Girls Service
Kesar Bagh Call Girl Price 9548273370 , Lucknow Call Girls ServiceKesar Bagh Call Girl Price 9548273370 , Lucknow Call Girls Service
Kesar Bagh Call Girl Price 9548273370 , Lucknow Call Girls Servicemakika9823
 
Russian Call Girl Brookfield - 7001305949 Escorts Service 50% Off with Cash O...
Russian Call Girl Brookfield - 7001305949 Escorts Service 50% Off with Cash O...Russian Call Girl Brookfield - 7001305949 Escorts Service 50% Off with Cash O...
Russian Call Girl Brookfield - 7001305949 Escorts Service 50% Off with Cash O...narwatsonia7
 
VIP Call Girls Pune Vrinda 9907093804 Short 1500 Night 6000 Best call girls S...
VIP Call Girls Pune Vrinda 9907093804 Short 1500 Night 6000 Best call girls S...VIP Call Girls Pune Vrinda 9907093804 Short 1500 Night 6000 Best call girls S...
VIP Call Girls Pune Vrinda 9907093804 Short 1500 Night 6000 Best call girls S...Miss joya
 
Artifacts in Nuclear Medicine with Identifying and resolving artifacts.
Artifacts in Nuclear Medicine with Identifying and resolving artifacts.Artifacts in Nuclear Medicine with Identifying and resolving artifacts.
Artifacts in Nuclear Medicine with Identifying and resolving artifacts.MiadAlsulami
 
Housewife Call Girls Hoskote | 7001305949 At Low Cost Cash Payment Booking
Housewife Call Girls Hoskote | 7001305949 At Low Cost Cash Payment BookingHousewife Call Girls Hoskote | 7001305949 At Low Cost Cash Payment Booking
Housewife Call Girls Hoskote | 7001305949 At Low Cost Cash Payment Bookingnarwatsonia7
 
Russian Call Girls in Bangalore Manisha 7001305949 Independent Escort Service...
Russian Call Girls in Bangalore Manisha 7001305949 Independent Escort Service...Russian Call Girls in Bangalore Manisha 7001305949 Independent Escort Service...
Russian Call Girls in Bangalore Manisha 7001305949 Independent Escort Service...narwatsonia7
 
Call Girls Service Pune Vaishnavi 9907093804 Short 1500 Night 6000 Best call ...
Call Girls Service Pune Vaishnavi 9907093804 Short 1500 Night 6000 Best call ...Call Girls Service Pune Vaishnavi 9907093804 Short 1500 Night 6000 Best call ...
Call Girls Service Pune Vaishnavi 9907093804 Short 1500 Night 6000 Best call ...Miss joya
 
VIP Call Girls Indore Kirti 💚😋 9256729539 🚀 Indore Escorts
VIP Call Girls Indore Kirti 💚😋  9256729539 🚀 Indore EscortsVIP Call Girls Indore Kirti 💚😋  9256729539 🚀 Indore Escorts
VIP Call Girls Indore Kirti 💚😋 9256729539 🚀 Indore Escortsaditipandeya
 

Recently uploaded (20)

Call Girls Service in Bommanahalli - 7001305949 with real photos and phone nu...
Call Girls Service in Bommanahalli - 7001305949 with real photos and phone nu...Call Girls Service in Bommanahalli - 7001305949 with real photos and phone nu...
Call Girls Service in Bommanahalli - 7001305949 with real photos and phone nu...
 
Russian Call Girls Chickpet - 7001305949 Booking and charges genuine rate for...
Russian Call Girls Chickpet - 7001305949 Booking and charges genuine rate for...Russian Call Girls Chickpet - 7001305949 Booking and charges genuine rate for...
Russian Call Girls Chickpet - 7001305949 Booking and charges genuine rate for...
 
Call Girls Yelahanka Just Call 7001305949 Top Class Call Girl Service Available
Call Girls Yelahanka Just Call 7001305949 Top Class Call Girl Service AvailableCall Girls Yelahanka Just Call 7001305949 Top Class Call Girl Service Available
Call Girls Yelahanka Just Call 7001305949 Top Class Call Girl Service Available
 
Call Girls Colaba Mumbai ❤️ 9920874524 👈 Cash on Delivery
Call Girls Colaba Mumbai ❤️ 9920874524 👈 Cash on DeliveryCall Girls Colaba Mumbai ❤️ 9920874524 👈 Cash on Delivery
Call Girls Colaba Mumbai ❤️ 9920874524 👈 Cash on Delivery
 
Call Girls Service Chennai Jiya 7001305949 Independent Escort Service Chennai
Call Girls Service Chennai Jiya 7001305949 Independent Escort Service ChennaiCall Girls Service Chennai Jiya 7001305949 Independent Escort Service Chennai
Call Girls Service Chennai Jiya 7001305949 Independent Escort Service Chennai
 
Sonagachi Call Girls Services 9907093804 @24x7 High Class Babes Here Call Now
Sonagachi Call Girls Services 9907093804 @24x7 High Class Babes Here Call NowSonagachi Call Girls Services 9907093804 @24x7 High Class Babes Here Call Now
Sonagachi Call Girls Services 9907093804 @24x7 High Class Babes Here Call Now
 
Call Girls Horamavu WhatsApp Number 7001035870 Meeting With Bangalore Escorts
Call Girls Horamavu WhatsApp Number 7001035870 Meeting With Bangalore EscortsCall Girls Horamavu WhatsApp Number 7001035870 Meeting With Bangalore Escorts
Call Girls Horamavu WhatsApp Number 7001035870 Meeting With Bangalore Escorts
 
Hi,Fi Call Girl In Mysore Road - 7001305949 | 24x7 Service Available Near Me
Hi,Fi Call Girl In Mysore Road - 7001305949 | 24x7 Service Available Near MeHi,Fi Call Girl In Mysore Road - 7001305949 | 24x7 Service Available Near Me
Hi,Fi Call Girl In Mysore Road - 7001305949 | 24x7 Service Available Near Me
 
High Profile Call Girls Jaipur Vani 8445551418 Independent Escort Service Jaipur
High Profile Call Girls Jaipur Vani 8445551418 Independent Escort Service JaipurHigh Profile Call Girls Jaipur Vani 8445551418 Independent Escort Service Jaipur
High Profile Call Girls Jaipur Vani 8445551418 Independent Escort Service Jaipur
 
Low Rate Call Girls Pune Esha 9907093804 Short 1500 Night 6000 Best call girl...
Low Rate Call Girls Pune Esha 9907093804 Short 1500 Night 6000 Best call girl...Low Rate Call Girls Pune Esha 9907093804 Short 1500 Night 6000 Best call girl...
Low Rate Call Girls Pune Esha 9907093804 Short 1500 Night 6000 Best call girl...
 
Call Girl Coimbatore Prisha☎️ 8250192130 Independent Escort Service Coimbatore
Call Girl Coimbatore Prisha☎️  8250192130 Independent Escort Service CoimbatoreCall Girl Coimbatore Prisha☎️  8250192130 Independent Escort Service Coimbatore
Call Girl Coimbatore Prisha☎️ 8250192130 Independent Escort Service Coimbatore
 
Kesar Bagh Call Girl Price 9548273370 , Lucknow Call Girls Service
Kesar Bagh Call Girl Price 9548273370 , Lucknow Call Girls ServiceKesar Bagh Call Girl Price 9548273370 , Lucknow Call Girls Service
Kesar Bagh Call Girl Price 9548273370 , Lucknow Call Girls Service
 
Russian Call Girl Brookfield - 7001305949 Escorts Service 50% Off with Cash O...
Russian Call Girl Brookfield - 7001305949 Escorts Service 50% Off with Cash O...Russian Call Girl Brookfield - 7001305949 Escorts Service 50% Off with Cash O...
Russian Call Girl Brookfield - 7001305949 Escorts Service 50% Off with Cash O...
 
VIP Call Girls Pune Vrinda 9907093804 Short 1500 Night 6000 Best call girls S...
VIP Call Girls Pune Vrinda 9907093804 Short 1500 Night 6000 Best call girls S...VIP Call Girls Pune Vrinda 9907093804 Short 1500 Night 6000 Best call girls S...
VIP Call Girls Pune Vrinda 9907093804 Short 1500 Night 6000 Best call girls S...
 
Escort Service Call Girls In Sarita Vihar,, 99530°56974 Delhi NCR
Escort Service Call Girls In Sarita Vihar,, 99530°56974 Delhi NCREscort Service Call Girls In Sarita Vihar,, 99530°56974 Delhi NCR
Escort Service Call Girls In Sarita Vihar,, 99530°56974 Delhi NCR
 
Artifacts in Nuclear Medicine with Identifying and resolving artifacts.
Artifacts in Nuclear Medicine with Identifying and resolving artifacts.Artifacts in Nuclear Medicine with Identifying and resolving artifacts.
Artifacts in Nuclear Medicine with Identifying and resolving artifacts.
 
Housewife Call Girls Hoskote | 7001305949 At Low Cost Cash Payment Booking
Housewife Call Girls Hoskote | 7001305949 At Low Cost Cash Payment BookingHousewife Call Girls Hoskote | 7001305949 At Low Cost Cash Payment Booking
Housewife Call Girls Hoskote | 7001305949 At Low Cost Cash Payment Booking
 
Russian Call Girls in Bangalore Manisha 7001305949 Independent Escort Service...
Russian Call Girls in Bangalore Manisha 7001305949 Independent Escort Service...Russian Call Girls in Bangalore Manisha 7001305949 Independent Escort Service...
Russian Call Girls in Bangalore Manisha 7001305949 Independent Escort Service...
 
Call Girls Service Pune Vaishnavi 9907093804 Short 1500 Night 6000 Best call ...
Call Girls Service Pune Vaishnavi 9907093804 Short 1500 Night 6000 Best call ...Call Girls Service Pune Vaishnavi 9907093804 Short 1500 Night 6000 Best call ...
Call Girls Service Pune Vaishnavi 9907093804 Short 1500 Night 6000 Best call ...
 
VIP Call Girls Indore Kirti 💚😋 9256729539 🚀 Indore Escorts
VIP Call Girls Indore Kirti 💚😋  9256729539 🚀 Indore EscortsVIP Call Girls Indore Kirti 💚😋  9256729539 🚀 Indore Escorts
VIP Call Girls Indore Kirti 💚😋 9256729539 🚀 Indore Escorts
 

Apendicitis 2

  • 1. 1121 PRINCIPLES Background The appendix was once considered a vestigial organ; however, it is currently theorized that it serves as a repository for commensal bacteria that assist in normal digestive processes and may allow for recolonization of intestinal flora in times of enteric bacterial destruction. Phylogenetic studies have supported the appendix as likely having a so-called positive fitness value during mammalian evolution, whereas recent clinical research studies have demon- strated a possible increased risk of clostridial infections in patients who have had prior appendectomies.1-3 Appendicitis is the most common cause of acute abdomi- nal pain requiring operative intervention in patients younger than 50 years. It is the most common nonobstetric abdominal emergency in pregnant females, usually occurring in the second trimester. Risk factors for appendicitis include white ethnicity, male gender, and young age (69% of cases occur in patients <30 years). Although males have an increased risk of having appen- dicitis (1.4 : 1), females have an almost twofold increased risk of undergoing an appendectomy, which is partially related to the fact that women have gynecologic conditions that frequently mimic appendicitis. Anatomy and Physiology The vermiform appendix is a blind-ended tube that originates from the cecum, approximately 3 cm from the ileocecal valve. It is considered part of the cecum and has the same histologic arrangement as the large intestine. A unique aspect of the appen- dix is the large masses of lymphoid tissue in the mucosa and submucosa. Although it has an average length of 8 to 10 cm, the appendix may be more than 20 cm in length, thereby allowing it to traverse into the left lower or right upper quadrants of the abdomen. The average diameter of the appendix ranges from 6 to 11 mm; thus, appendiceal diameter alone, in the absence of other radiologic findings, often does not imply appendicitis. Finally, a normal appendix has an average wall thickness of 1.5 mm. The appendix maintains afferent sensory fibers that follow the sympathetic innervation and enter the spinal cord at the 10th thoracic level (T10). It is these nerves that carry the sensation of pain and result in the periumbilical discomfort associated with early appendicitis. There are three important anatomic features of the appendix that determine the site of the patient’s pain and tenderness when the organ is inflamed: (1) the location of the origin of the appen- dix off the cecum; (2) the course the appendix takes from this origin; and (3) the length of the appendix (as described above; Fig. 83.1). All these features are variable, resulting in a wide range of symptoms and signs, which often creates difficulty with the clinical diagnosis of appendicitis. Although the location of the origin of the appendix is generally thought to be positioned at McBurney’s point (exactly between an 1.5 to 2 inches from the right anterior superior iliac spine on a straight line drawn to the umbilicus), the base may be as much as 10 cm away from this site. In fact, only 40% of patients have the base of their appendix within 3m of McBurney’s point, with 36% of patients having the base more than 5m away. Next, there is significant variation involving the course of the appendix from its cecal origin. The frequency with which the appendix is found in various loca- tions is illustrated in Fig. 83.2. Finally, in rare cases, patients may present with left lower quadrant pain or right flank pain due to a very long appendix traveling into the left lower abdomen or a retrocecal location extending into the retroperitoneum, respectively. Pathophysiology The cause of appendicitis is rooted in obstruction of the appen- diceal lumen. The underlying pathophysiology is progressive in nature and best understood in a stepwise fashion—appendiceal obstruction prevents egress of mucus and bacteria from the appendix; continued mucous production and bacterial prolifera- tion result in luminal distention, which stimulates the T10 visceral afferent nerves, creating periumbilical pain typically lasting 4 to 6 hours. Intraluminal pressure eventually exceeds local capillary pressure in the appendiceal wall, preventing arterial perfusion and resulting in tissue ischemia and inflammation; ischemia and inflammation compromise the integrity of the appendiceal wall bacteria and then invade the appendiceal wall. This causes trans- mural inflammation that extends into the surrounding tissues (peritoneal, ileocecal, and pelvic areas), resulting in somatic local- ized pain, typically focused in the right lower quadrant. If this process continues, the appendix becomes necrotic and perforates, releasing enteric contents into the peritoneum and resulting in peritonitis and, typically, diffuse abdominal pain. The length of time from the onset of symptoms to perforation is highly variable. Although obstruction of the appendiceal lumen is thought to be a common inciting factor in appendicitis, the cause of this obstruction is variable and, in many cases, a source of obstruction is not discovered on imaging or pathology. Fecaliths (hard stools) are the most common cause of obstruction in nonperforated appendicitis (65%), followed by appendicoliths (calcified depos- its) and lymphoid hyperplasia (primary or secondary to an enteric infection). Other causes of obstruction are rare; these include fecal stasis, foreign bodies (eg, vegetable matter, inspissated barium), tumors, and intestinal parasites. There are two additional pathologic processes associated with appendicitis. First, so-called tip appendicitis is appendiceal inflammation localized to the distal end of the appendix. The clinical significance of this disorder is that it may be missed on imaging due to the limited extent of disease and the lack of classic findings associated with appendicitis; it has been demonstrated to contribute to the false-negative rate associated with computed tomography (CT) imaging. Second, stump appendicitis is a very rare entity that results from inflammation of the appendiceal remnant that may persist after the appendix has been removed surgically. Timing of its occurrence varies; it has been diagnosed C H A P T E R 83  Acute Appendicitis Michael Alan Cole  |  Robert David Huang Descargado para Francisco Medina Anderson (fz.medina@hotmail.com) en Organización de Estados Iberoamericanos - Remote Access de ClinicalKey.es por Elsevier en julio 07, 2017. Para uso personal exclusivamente. No se permiten otros usos sin autorización. Copyright ©2017. Elsevier Inc. Todos los derechos reservados.
  • 2. 1122 PART III  Medicine and Surgery  |  SECTION Five  Gastrointestinal System treatment, and disposition. This approach should focus on the history and physical examination findings that have the greatest predictive values. History The history and review of systems provide insight into the patient’s symptoms and help determine alternative diagnoses. A prior history of similar symptoms suggests an alternative diagnosis, because appendicitis is an acute illness. Furthermore, because the pathophysiology of appendicitis is a progressive process,a patient’s symptoms typically worsen over the course of the illness until perforation of the appendix occurs. At this point, the patient may receive some temporary relief due to a decrease of intraluminal pressure, but will subsequently become very ill from the resultant peritonitis. See Table 83.1 for a list common symptoms and their value in predicting the likelihood of appendicitis. Duration of symptoms is variable and often is not useful in the assessment of appendicitis. When considering features that exclude appendicitis, findings that decrease the likelihood of appendicitis include absence of RLQ pain and a history of similar pain in the past. Nevertheless, these findings in isolation do not exclude appendicitis as a pos- sible diagnosis and are best used as part of a comprehensive clini- cal decision making process. Physical Examination All patients with abdominal pain should be fully disrobed, and female patients should ideally be placed in a room in which a pelvic examination can be performed (see Table 83.1). Classic, eponymous examination maneuvers for appendicitis have overall poor sensitivity but, if present, have a modest predictive value (Table 83.2). Although McBurney’s point tenderness has a low correlation with appendiceal location and is not highly sensitive for appendicitis, tenderness at this location does have a modest predictive value for appendicitis. A genitourinary examination should be performed to assess for testicular pathology or hernias in males and pelvic pathology in females. Cervical motion tenderness (CMT) is not specific for pelvic pathology and is noted in 28% of female patient with appendicitis. A rectal examination contributes little toward the assessment of appendicitis and is not routinely recommended.6 DIFFERENTIAL DIAGNOSIS There are many diagnoses that mimic appendicitis; alternatively, appendicitis may present atypically and must be considered in all between 4 days and up to 50 years after the initial appendectomy.4 CLINICAL FEATURES Typical presentations of appendicitis often allow an experienced provider to hone in on a diagnosis in a rather rapid fashion; unfortunately, typical findings are the exception and not the rule. For historical purposes, the constellation findings classically asso- ciated with acute appendicitis include younger age, epigastric pain that migrates to the right lower quadrant (RLQ), and tenderness of the RLQ. However, the combination of these findings occurs in less than 50% of patients with acute appendicitis thus limiting their use in clinical decision making.4a Missed acute appendicitis is one of the most common causes of litigation surrounding emer- gency medicine, which speaks to the challenges in diagnosing this illness in the presence of often ambiguous symptoms.5 No one element of the history or physical examination can reliably be used to diagnose or exclude appendicitis. Therefore, focusing on a single finding may lead to misdiagnosis. Rather, a comprehensive approach using multiple elements of the history, physical examination, and laboratory data should be used to risk- stratify patients to make informed decisions regarding imaging, Fig. 83.1.  This figure demonstrates the variation in location of the base of the appendix and its course within the peritoneum. McBurney’s point classically represents the point of maximal tenderness in appendicitis; however, depending on the path, length, and degree of inflammation of the appendix, the true point of maximal tenderness is highly variable. Subcecal (2%) Pelvic (32%) Preilial (1%) Postileal (0.4%) Retrocecal (64%) The origin of the appendix can vary as much as 10 cm from McBurney’s point. TABLE 83.1  Predictive Value of Common Symptoms and Signs in Diagnosing Appendicitis FEATURE MODERATELY USEFUL MILDLY USEFUL NOT USEFUL Historical features RLQ pain Migration of pain to the RLQ Presence of pain prior to vomiting No history of prior similar pain Vomiting Male gender Pain worsened when driving over speed bumps34 Anorexia Nausea Pain worse with cough or movement Physical examination features35 RLQ tenderness Abdominal wall rigidity Pain focused at McBurney’s point Rebound tenderness Guarding Temperature > 38.3°C (101°F) Percussion tenderness Psoas sign Rectal examination Increased skin temperature RLQ, Right lower quadrant. Adapted from Laurell H, Hansson L-E, Gunnarsson U. Manifestations of acute appendicitis: a prospective study on acute abdominal pain. Dig Surg 30:198–206, 2013. Descargado para Francisco Medina Anderson (fz.medina@hotmail.com) en Organización de Estados Iberoamericanos - Remote Access de ClinicalKey.es por Elsevier en julio 07, 2017. Para uso personal exclusivamente. No se permiten otros usos sin autorización. Copyright ©2017. Elsevier Inc. Todos los derechos reservados.
  • 3. 1123CHAPTER 83  Acute Appendicitis comprehensive assessment of the patient’s condition and further risk-stratify the patient for treatment and disposition purposes. White Blood Cell Count A patient’s white blood cell (WBC) count does not by itself have the sensitivity, specificity, or predictive value necessary to be clini- cally useful in diagnosing or excluding appendicitis. An elevated WBC count (>10,000–12,000/mm3 ) has a sensitivity of 62% to 85%, specificity of 32% to 82%, positive LR of 1.59 to 2.7, and negative LR of 0.25 to 0.46. Even in a subgroup analysis of patients presenting with abdominal pain, not just those with right lower quadrant pain. Table 83.3 lists the most common differen- tial diagnoses for appendicitis. DIAGNOSTIC TESTING Laboratory Data Laboratory data should not be viewed as diagnostic for appendi- citis. Rather, it should be used in association with the patient’s clinical history and physical examination to formulate a more Fig. 83.2.  Suggested clinical management pathway for emergency department patients with possible appendicitis. Gen., General; OB, obstetrician. Develop pretest probability for appendicitis based on history, physical examination, and laboratory data Treat symptomatically Low risk • Consider/treat alternative diagnoses • Discharge with precautionary instructions High riskPregnant No Negative/ nondiagnostic Positive Positive Negative/ nondiagnostic Positive Negative Negative Positive Yes Consult OB & Gen. Surgery Moderate risk* No Yes No Yes Operative appendix removal within 12 hours of diagnosis Admit for continued IV antibiotics, serial examinations and observation NoYes CT with IV contrast (no enteric contrast) Is the patient still symptomatic Diagnosis of appendicitis is made Administer IV antibiotics Admit for observation and symptomatic therapy • Consider/treat alternative diagnoses • Discharge with precautionary instructions Admit for observation and symptomatic therapy Surgical consultation Is the patient a candidate for conservative management Is the patient still symptomatic? MRI with no IV contrast (+/- enteric contrast) Consider graded compression ultrasound** Graded compression ultrasound *In moderate pre-test probability patients, the provider may consider admission for serial examinations or discharge in select cases. **In pediatric patients, graded compression ultrasound should always be the first imaging test performed. Descargado para Francisco Medina Anderson (fz.medina@hotmail.com) en Organización de Estados Iberoamericanos - Remote Access de ClinicalKey.es por Elsevier en julio 07, 2017. Para uso personal exclusivamente. No se permiten otros usos sin autorización. Copyright ©2017. Elsevier Inc. Todos los derechos reservados.
  • 4. 1124 PART III  Medicine and Surgery  |  SECTION Five  Gastrointestinal System Urinalysis Urinalysis demonstrates pyuria, hematuria, and/or bacteria in up to 48% of patients with appendicitis. These abnormalities are due to the inflamed appendix abutting the ureter, with resultant ure- teral inflammation. Nevertheless, findings on urinalysis of more than 30 red blood cells (RBCs)/high-power field or more than 20 WBCs/high-power field are more consistent with urinary tract infections than appendicitis. Other Laboratory Tests A serum or urine pregnancy test is recommended for any female of childbearing age with abdominal pain. A basic metabolic panel, liver function tests, and lipase level should be obtained for patients with suspected appendicitis to assess for electrolyte derangements and alternative causes of abdominal pain. Procalcitonin does not currently play a role in the diagnosis of appendicitis due to its poor predictive value.8 The polymorphonuclear count, in isola- tion, has no clinical value in the assessment of appendicitis. Imaging Tests General Principles The decision to pursue imaging is based on the provider’s clinical assessment, which combines the patient’s history, examination, and laboratory data to decide on the likelihood of appendicitis. If the likelihood is low (and other significant disease processes have been excluded), the patient may be discharged from the emergency department (ED) or observed with serial examinations in an observation unit (see Chapter e6) or inpatient setting. However, if there is a concern for appendicitis, imaging should be carried out. Currently, patients rarely undergo surgical removal of the appendix based on clinical features alone. The negative appendec- tomy rate—the number of normal appendices that are surgically removed—is far lower when imaging is used.11 Nevertheless, in rare cases of young men with a classic presentation, the decision to perform an appendectomy in the absence of imaging may be pursued at the surgeon’s discretion. Radiography Due to their poor sensitivity and specificity, routine radiographs are of no clinical value in the evaluation of appendicitis. The only value of radiographs is to assess for other causes of the patient’s symptoms, such as bowel obstruction or bowel perforation. However, an ileus mimicking bowel obstruction may occur in appendicitis due to peritoneal inflammation, and advanced appendicitis may perforate, resulting in intraperitoneal air on TABLE 83.2  Common Maneuvers and Physical Findings Associated With Appendicitis and Their Predictive Valuesa MANEUVER DESCRIPTION SENSITIVITY AND SPECIFICITY (%) Iliopsoas (psoas) sign Increased abdominal pain with patient lying on left side while provider passively extends the patient’s right leg at the hip with both knees extended Sensitivity: 13–42 Specificity: 79–95 Rovsing’s sign Abdominal pain in the RLQ while palpating the left lower quadrant Sensitivity: 7–68 Specificity: 58–96 Obturator sign Increased abdominal pain in the supine patient as the provider internally and externally rotates the right leg as it is flexed at the hip Sensitivity: 8 Specificity: 94 a Overall poor sensitivity decreases the value of these findings. However, if found, these signs moderately increase the likelihood of having appendicitis. RLQ, Right lower quadrant. TABLE 83.3  Differential Diagnosis in Appendicitis ALL PATIENTS FEMALE PATIENTS PEDIATRIC PATIENTS Nonspecific abdominal pain Gastroenteritis Epiploic appendigitis Ureterolithiasis, nephrolithiasis Inflammatory bowel disease Ileus or bowel obstruction Intestinal perforation Testicular torsion (males) Ectopic pregnancy Ovarian torsion Pelvic inflammatory disease Ovarian cyst Henoch-Schönlein purpura Mesenteric lymphadenitis Meckel’s diverticulum increasingly high cutoff values for the WBC count (eg, >15,000 or 20,000/mm3 ), it is still not significant enough to be used in clinical practice to diagnose or exclude acute appendicitis.7,8 C-Reactive Protein The C-reactive protein (CRP) level is a nonspecific, systemic inflammatory marker synthesized by the liver. It has a poor pre- dictive value in diagnosing or excluding acute appendicitis.8 An elevated CRP (>8–10 mg/L) has a sensitivity of 65% to 85%, specificity of 32% to 87%, positive LR of 1.59 to 4.2, and negative LR of 0.11 to 3.2.8,9 Some studies have suggested that CRP may be useful for predicting the severity of appendicitis and likelihood of complications; however, its value in diagnosing appendicitis lies in combining CRP with the WBC (see below).10 Combined Inflammatory Markers The American College of Emergency Physicians clinical policy on patients with suspected appendicitis states that the combination of a WBC more than 10,000/mm3 and CRP more than 8 mg/L has a positive likelihood ratio of 23 and a negative likelihood ratio of 0.03.These combined laboratory findings offer the greatest impact when excluding appendicitis in patients with a low pretest prob- ability of the disease. Although more research is needed, based on the best available evidence, we recommend using the combination of low WBC (<10,000/mm3 ) and CRP (<8 mg/L) in patients deemed to be low risk for appendicitis, based on the provider’s clinical assessment, to exclude appendicitis. Alternatively, we do not believe that there are sufficient data to establish the diagnosis of appendicitis conclusively based on the combination of elevated WBC and CRP levels; rather, these should only assist in the deci- sion making process regarding the need for imaging. Descargado para Francisco Medina Anderson (fz.medina@hotmail.com) en Organización de Estados Iberoamericanos - Remote Access de ClinicalKey.es por Elsevier en julio 07, 2017. Para uso personal exclusivamente. No se permiten otros usos sin autorización. Copyright ©2017. Elsevier Inc. Todos los derechos reservados.
  • 5. 1125CHAPTER 83  Acute Appendicitis pelvic US is an important study to help determine ovarian pathol- ogy or tuboovarian abscesses. This should be performed before CT imaging in an attempt to elucidate an alternative diagnosis and may be completed simultaneously with a graded compression US to assess for appendicitis. Computed Tomography CT of the abdomen and pelvis is considered the test of choice for definitive assessment of possible appendicitis in nonpregnant patients. It demonstrates an overall sensitivity of 94% to 100% and specificity of 91% to 99%, with a positive LR of 9.29 to 13.3, negative LR of 0.1 to 0.09, and positive predictive value of 95% to 97%.13 CT is accurate and consistent in diagnosing appendicitis and decreases the negative appendectomy rate. CT is readily avail- able in most hospitals, can be performed in a rapid fashion, is not operator-dependent, can be interpreted by most radiologists and surgeons, and has a greater likelihood of finding an alternative diagnosis (vs. US; Figs. 83.5 and 83.6). abdominal radiographs. Therefore, care must be taken to make the final diagnosis based on radiographic findings, although intra- peritoneal air often expedites the patient’s disposition to the oper- ating room. Graded-Compression Ultrasound Within the medical community, there is a growing awareness of the risks associated with ionizing radiation, and efforts are being made to use methods of diagnosis that reduce or eliminate these risks.12 Graded compression ultrasound (US) is an imaging tool commonly used in evaluating patients for appendicitis. It is a diagnostic technique in which steady pressure is applied with the US probe to the abdomen to reduce bowel gas and collapse normal bowel to promote visualization of the appendix. Studies involving graded compression US for the diagnosis of appendicitis have reported sensitivities of 75% to 90%, specificities of 83% to 95%, positive LRs of 4.5 to 5.8, and negative LRs of 0.19 to 0.27, with an average positive predictive value of 90%.13 Table 83.4 lists US criteria for the diagnosis of appendicitis. The benefits in using US for the diagnosis of appendicitis include decreased cost relative to other imaging modalities, lack of ionizing radiation exposure, and decreased time to diagnosis. Limitations of US use include decreased specificity and increased pain due to the transducer pressure needed for the graded com- pression process. Most importantly, a number of US examinations cannot visualize the appendix (ie, nondiagnostic) for a number of reasons, including lack of operator experience, patient factors (eg, obesity), superimposed bowel gas, or atypically located appen- dix.14 In cases with nondiagnostic US findings, the patient typi- cally requires further imaging with CT (or magnetic resonance imaging [MRI] in pregnancy) or admission for observation and serial examinations. Ultrasound is most useful in children, for whom the risks of ionizing radiation are greatest, and rates of overweight and obese individuals are lower than adults and preg- nant females (Figs. 83.3 and 83.4). A distinction must be made between radiology-based US and bedside (point of care) US examination performed by an emer- gency clinician. Recent studies have demonstrated that bedside US is not as effective at diagnosing appendicitis, with a sensitivity for diagnosis of 60% to 70%, with specificities of 94% to 98%.15,16 Finally, in women with CMT, masses found on pelvic examina- tion, or concern for a gynecologic cause of the patient’s symptoms, TABLE 83.4  Diagnostic Criteria for Appendicitis on Imaging ULTRASOUND COMPUTED TOPOGRAPHY MAGNETIC RESONANCE IMAGING The first two criteria below must be fulfilled: Not all criteria listed below need to be fulfilled but the combination and severity of these findings contribute to a diagnosis: Not all criteria listed need to be fulfilled but the combination and severity of these findings contribute to a diagnosis: • Appendiceal diameter > 6–7 mma Appendiceal diameter (>6 mm with surrounding inflammation or >8 mm without such changes) Appendiceal diameter > 7 mm • Noncompressible appendix Appendiceal circumferential wall thickening >2 mm with mural enhancement (sign of inflammation) Appendiceal circumferential wall thickening > 2 mm Fat stranding (hyperechoic signals associated with periappendiceal inflammation) (secondary finding) and peritoneal fluid Calcified appendicolith Signs of inflammation adjacent to the appendix, such as fat stranding or phlegmon formation Peritoneal fluid surrounding the appendix (secondary finding) Signs of periappendiceal inflammation (eg, fat stranding, clouding of the adjacent mesentery) Presence of an abscess or a fluid filled appendix a It is important to note that the diameter of a normal nondiseased appendix may be up to 11 mm, so the other findings of appendicitis must be factored in when making the diagnosis of appendicitis on CT or MRI. Due to the graded compression technique used in ultrasound, there is more certainty regarding diagnostic criteria for appendiceal diameter. Fig. 83.3.  Ultrasound image of appendicitis in an 8-year-girl. Note the dilated noncompressible appendix (thin arrows) and the presence of a fecalith, with posterior acoustic shadowing (thick arrow). (Courtesy Dr. Michael Cole, with permission.) Descargado para Francisco Medina Anderson (fz.medina@hotmail.com) en Organización de Estados Iberoamericanos - Remote Access de ClinicalKey.es por Elsevier en julio 07, 2017. Para uso personal exclusivamente. No se permiten otros usos sin autorización. Copyright ©2017. Elsevier Inc. Todos los derechos reservados.
  • 6. 1126 PART III  Medicine and Surgery  |  SECTION Five  Gastrointestinal System To this end, there have been recent studies of low-dose CT protocols for the diagnosis of appendicitis. These low-dose pro- tocols decrease the average dose to approximately 2 mSv, with no detriment in the negative appendectomy rate. However, there is less diagnostic certainty by radiologists about the diagnosis of appendicitis with these studies. These are relatively new protocols that show promise but require more studies before they can be universally adopted.17,18 Table 83.4 lists CT findings diagnostic of appendicitis. In some cases, the appendix cannot be visualized. In these cases, if CT demonstrates no findings of inflammation in the RLQ, it has been found that appendicitis is unlikely. However, patients with low amounts of intra-abdominal body fat may not display secondary signs of inflammation; consequently, these patients may lack this important marker of appendicitis on CT imaging, leading to false- negative study results. The term tip appendicitis refers to obstruc- tion and inflammation limited to the distal tip of the appendix and is a subtle finding on CT that is a common cause of false- negative interpretation.19 To assess for appendicitis, CT should be performed with IV contrast only. Enteric contrast of any type, oral or rectal, contrib- utes little to the assessment of appendicitis. In addition, studies have demonstrated that non–contrast-enhanced CT has accept- able accuracy in diagnosing appendicitis. Furthermore, according to the American College of Radiology’s appropriateness criteria for imaging suspected appendicitis, CT imaging with or without IV contrast are acceptable imaging modalities, with the use of enteric contrast being deferred to institutional preference. There- fore, if there are contraindications to IV contrast, there should be little hesitation to move forward with non–contrast-enhanced CT for the evaluation of appendicitis.17 Magnetic Resonance Imaging When considering the evaluation for appendicitis, current evi- dence supports the use of MRI for assessment in pregnant females if US is nondiagnostic. MRI has the advantage of not using ion- izing radiation and is not operator-dependent. However, its use is limited by its increased cost, increased time required to acquire images,limited availability,and need for the radiologist or surgeon The greatest disadvantage of CT is the ionizing radiation. A CT scan of the abdomen exposes the patient to an average dose of ionizing radiation equivalent to 8 examination 10 mSv. To put this in perspective, the average ionizing radiation dose associated with an abdominal x-ray is 0.7 mSv, and the average dose associated with coronary angioplasty is 15 mSv. An abdominal CT carries an excess risk of fatal cancer of 1 in 2000, a value that is even greater in children. However, this value must be tempered by the fact that the general population has a lifetime risk of being diagnosed with cancer of 1 in 3. The risk of radiation increases conversely with age, with children and fetuses having the greatest risk of adverse outcomes of radiation due to their smaller body habitus, more rapidly developing cells, and increased incubation time for genetic mutations to manifest. Fig. 83.5.  CT scan with typical findings of acute appendicitis. thick arrow, thin arrows, dashed line arrow. Fig. 83.6.  Oral contrast CT scan showing discrete abscess from appen- diceal perforation, with periappendiceal fat streaking. (Courtesy Jefferson Radiology, Avon, CT.) Fat stranding Discrete abscess from perforated appendix Fig. 83.4.  Graded compression ultrasound scan demonstrating a dilated noncompressible appendix (thin arrow) representing appendicitis. Descargado para Francisco Medina Anderson (fz.medina@hotmail.com) en Organización de Estados Iberoamericanos - Remote Access de ClinicalKey.es por Elsevier en julio 07, 2017. Para uso personal exclusivamente. No se permiten otros usos sin autorización. Copyright ©2017. Elsevier Inc. Todos los derechos reservados.
  • 7. 1127CHAPTER 83  Acute Appendicitis IV fluids, antipyretics, and antibiotics (see below). The patient’s pain and nausea should be treated with parenteral opiate analgesia and antiemetics, respectively. There have been a number of good- quality studies that support the concept that opiate analgesia does not negatively affect a patient’s abdominal examination when the patient has an abdominal condition that requires surgery.26,27 Therefore, parenteral opiate analgesia should not be withheld from the patient unless there are contraindications to its use (eg, severe hypotension, allergies). In rare cases, acute appendicitis can cause severe sepsis or septic shock. Antibiotic Therapy Antibiotic therapy should be promptly administered on making the diagnosis of appendicitis or in patients with suspected appen- dicitis and severe sepsis or septic shock. The choice of antibiotics should include broad-spectrum gram-negative and anaerobic coverage. For nonperforated appendicitis, we recommend cipro- floxacin, 400 mg IV, and metronidazole (Flagyl), 500 mg IV; or ceftriaxone, 1g IV, and metronidazole, 500 mg IV; or ampicillin- sulbactam, 3g IV monotherapy. For perforated appendicitis, we recommend broader spectrum antibiotics, such as piperacillin- tazobactam, 3.375 to 4.5g IV, cefepime, 2 g IV, or imipenem- cilastatin, 500 mg IV. Methicillin-resistant Staphylococcus aureus (MRSA) coverage is not typically needed to treat appendicitis but may be considered if the patient has previously known MRSA colonization.28 Definitive Treatment Definitive treatment of acute appendicitis will depend on whether there are associated complications, and all decisions should be made in consultation with the surgical service. Nonperforated appendicitis with a well-circumscribed abscess should be treated with IV antibiotics and percutaneous drainage. Perforated appen- dicitis with or without abscess is treated with IV antibiotics and urgent operative intervention.28 Nonperforated appendicitis without abscess (ie,uncomplicated appendicitis) is traditionally treated with IV antibiotics and surgi- cal removal of the inflamed appendix. However, recent and his- torical data have demonstrated that conservative treatment of appendicitis with antibiotic therapy and a period of inpatient observation may be a viable treatment option for certain patients. There is historical precedence for nonoperative management of appendicitis,and recent studies have found that there may be value in risk-stratifying patients with appendicitis based on their CT findings. In appendicitis with low-risk features, antibiotic therapy with a period of inpatient observation is a feasible option.29,30 Features associated with failed conservative management include the presence of a fecalith, abscess, tumor, or fluid collection or appendiceal diameter of more than 1.1 cm.31 In patients with any of these features, operative intervention is preferred. A minority of patients treated conservatively may fail the inpa- tient observation period and still require surgery; a minority of those discharged after conservative treatment carry the risk of recurrence of appendicitis. However, with a negative appendec- tomy rate of 3.6% to 10% and a complication rate as high as 18%—including small bowel obstruction, adhesions, surgical site infection, and abscess formation—nonoperative care is an option worth considering.32 The decision regarding definitive treatment of acute appendicitis should be made in consultation with the surgical service and the risks and benefits of conservative treat- ment versus surgical intervention should be frankly discussed with the patient, surgeon, and emergency clinician. When the decision is made to proceed with surgical removal of the appendix, in uncomplicated appendicitis, delaying surgery up to 12 hours after diagnosis is made (eg, “waiting until the to be skilled in MRI scan interpretation. MRI demonstrates a sensitivity of 85% to 100%, specificity of 95% to 99.2, average positive predictive value of 92.4, and average negative predictive value of 99.7.20 Table 83.4 lists MRI criteria for the diagnosis of appendicitis. In pregnant patients, IV gadolinium contrast should not be used when evaluating for appendicitis due to potentially harmful effects on the fetus.21 Enteric contrast may be used at the discre- tion of the interpreting radiologist or per institutional protocol. Combined Imaging Pathways An imaging pathways that combine US and CT, in which abdomi- nopelvic CT is performed if the graded compression US is non- diagnostic or negative, have demonstrated combined sensitivities of 94% to 99%, specificities of 91% to 97.5%, and significant reductions in CT utilization.22,23 It has been projected that this pathway would save $547/patient in imaging costs and $25 million/year in aggregate by reducing imaging costs, unnecessary surgeries, and unnecessary hospitalizations, not to mention decreased radiation exposure.24 As institutions increase their experience with the use of US to diagnose appendicitis, we think that a combined US-CT pathway will gain acceptance and improve health care delivery. Interestingly, a so-called radiation-free imaging pathway that combines US and MRI, in which abdominopelvic MRI is per- formed if the US is nondiagnostic or negative, has been recently studied in the emergency pediatric population, with outcomes similar to those of the combined US-CT pathway. However, at this time, there is a paucity of sufficient data and lack of institutional resources to suggest the routine use of this approach.25 Summary of Imaging Methods Fig. 83.2 illustrates a suggested pathway regarding imaging. For nonpregnant patients, graded compression US may be first con- sidered. In nonpregnant females, a pelvic US may also be consid- ered to assess for pelvic pathology. The ability to visualize the appendix on US is institution-dependent, and the provider’s deci- sion to use US initially may depend on the institution’s level of experience with this modality. If the US studies are negative or nondiagnostic (ie, no appendix is visualized and no alternative pathology is noted), the patient may undergo CT imaging of the abdomen and pelvis with IV contrast (no PO contrast). An alter- native to CT imaging in low-risk cases with nondiagnostic US is admission for observation and serial examinations. If the patient is pregnant, graded compression and pelvic US should always be the initial studies of choice, followed by MRI of the abdomen without IV contrast in cases of nondiagnostic or negative US findings. If MRI is not available, and transfer to a facility with MRI capabilities is not feasible, then, after consulta- tion with a radiologist, general surgeon, and obstetrician, abdomi- nal CT scanning with IV contrast may be considered. However, in low-risk cases, admission for observation and serial examinations is an acceptable alternative. MANAGEMENT Supportive Care Decisions surrounding supportive care will depend on the patient’s condition and needs. Supportive care should be initiated prior to a definitive diagnosis and should continue until the patient leaves the ED. Patients should remain NPO. IV fluids (normal saline or lactated Ringer’s) may be administered to maintain hydration and support hypotensive patients. Systemic signs of infection are more common in perforated appendicitis and should be supported by Descargado para Francisco Medina Anderson (fz.medina@hotmail.com) en Organización de Estados Iberoamericanos - Remote Access de ClinicalKey.es por Elsevier en julio 07, 2017. Para uso personal exclusivamente. No se permiten otros usos sin autorización. Copyright ©2017. Elsevier Inc. Todos los derechos reservados.
  • 8. 1128 PART III  Medicine and Surgery  |  SECTION Five  Gastrointestinal System based on imaging or, rarely, clinical assessment alone. In this case, antibiotics should be initiated, surgical consultation should be obtained, and the patient should be admitted for operative inter- vention or, in select cases, IV antibiotics and observation. Based on clinical and laboratory assessment, the risk of appendicitis is low, and no imaging study was performed. In this case, the patient may be discharged home if he or she is reliable, has improved clinical status (ie, feels better), and understands the provider’s thought process and precautionary instructions. Alternatively, if these criteria are not met, the patient may be transferred to an observation unit or hospitalized for serial examinations. If the patient’s imaging results are inconclusive, or if they are negative but the patient is still symptomatic, the patient may be admitted for observation, symptomatic treatment, serial examinations, and kept NPO, although select patients in this category may still be discharged at the provider’s discretion. morning”) is acceptable and does not lead to worse clinical out- comes; one recent study has demonstrated that inpatient delay of appendectomy by up to 24 hours does not result in worse out- comes.33 Although limited in-hospital delay of operative care has not been shown to increase perforation risk or morbidity, delay by patients initially seeking care does increase the risk of perfora- tion and associated morbidity. Lack of insurance, male gender, and a greater number of comorbid conditions are factors associ- ated with increased perforation risk. The choice of laparoscopic versus open appendectomy is made by the surgeon; however, lapa- roscopic appendectomy has become the current method of choice. DISPOSITION There are three possible disposition pathways when a diagnosis of appendicitis is considered. A diagnosis of appendicitis is made • Appendicitis is a progressive illness caused by appendiceal luminal distention followed by appendiceal wall ischemia, transmural inflammation, and eventual perforation, with resultant peritonitis. • Clinical history, physical examination, and laboratory findings need to be combined to formulate a comprehensive assessment. No one finding can definitively diagnose or exclude appendicitis. • The most useful historical features in evaluating appendicitis are RLQ pain, pain preceding vomiting, and migration of pain to the RLQ. • The most useful physical findings in evaluating appendicitis are RLQ tenderness and rigidity. • Cervical motion tenderness is not specific for pelvic pathology and is found in up to 28% of females with appendicitis. • A rectal examination contributes little and should not be routinely performed in the evaluation of appendicitis. • The white blood cell count alone is neither sensitive nor specific for appendicitis and offers little in the evaluation of appendicitis. • When clinicians have a low pretest possibility for appendicitis, the combination of a WBC count below 10,000/mm3 and CRP level below 8 mg/L support the exclusion of appendicitis as a likely diagnosis. • Nonoperative management of acute appendicitis (IV antibiotics, admission) is gaining support. The patient should not have high-risk features (eg, presence of a fecalith, abscess, tumor, or fluid collection or appendiceal diameter >1.1 cm) and should be made aware of the risk of failed observation as an inpatient or recurrent appendicitis once discharged, both of which would then require surgical removal of the appendix. • Once the diagnosis of appendicitis is made, in-hospital delay of appendectomy of up to 12 hours has not demonstrated negative outcomes when compared to emergent operative care. KEY CONCEPTS The references for this chapter can be found online by accessing the accompanying Expert Consult website. Descargado para Francisco Medina Anderson (fz.medina@hotmail.com) en Organización de Estados Iberoamericanos - Remote Access de ClinicalKey.es por Elsevier en julio 07, 2017. Para uso personal exclusivamente. No se permiten otros usos sin autorización. Copyright ©2017. Elsevier Inc. Todos los derechos reservados.
  • 9. 1128.e1CHAPTER 83  Acute Appendicitis REFERENCES 1. Smith HF, Parker W, Kotzé SH, et al: Multiple independent appearances of the cecal appendix in mammalian evolution and an investigation of related ecological and anatomical factors. Comptes Rendus Palevol 12:339–354, 2013. 2. Im GY, Modayil RJ, Lin CT, et al: The appendix may protect against Clostridium difficile recurrence. Clin Gastroenterol Hepatol 9:1072–1077, 2011. 3. Clanton J, Subichin M, Drolshagen K, et al: Fulminant Clostridium difficile infection: an association with prior appendectomy? World J Gastrointest Surg 5:233–238, 2013. 4. Hendahewa R, Shekhar A, Ratnayake S: The dilemma of stump appendicitis—a case report and literature review. Int J Surg Case Rep 14:101–103, 2015. 4a.  Laméris W, van Randen A, Go PM, et al: Single and combined diagnostic value of clinical features and laboratory tests in acute appendicitis. Acad Emerg Med 16(9): 835–842, 2009. 5. Brown TW, McCarthy ML, Kelen GD, et al: An epidemiologic study of closed emer- gency department malpractice claims in a national database of physician malpractice insurers. Acad Emerg Med 17:553–560, 2010. 6. Takada T, Nishiwaki H, Yamamoto Y, et al: The role of digital rectal examination for diagnosis of acute appendicitis: a systematic review and meta-analysis. PLoS ONE 10:e0136996, 2015. 7. Howell JM, Eddy OL, Lukens TW, et al: Clinical policy: critical issues in the evaluation and management of emergency department patients with suspected appendicitis. Ann Emerg Med 55:71–116, 2010. 8. Yu C-W, Juan L-I, Wu M-H, et al: Systematic review and meta-analysis of the diag- nostic accuracy of procalcitonin, C-reactive protein and white blood cell count for suspected acute appendicitis. Br J Surg 100:322–329, 2013. 9. Farooqui W, Pommergaard H-C, Burcharth J, et al: The diagnostic value of a panel of serological markers in acute appendicitis. Scand J Surg 104:72–78, 2015. 10. Shindoh J, Niwa H, Kawai K, et al: Diagnostic power of inflammatory markers in predicting severity of appendicitis. Hepatogastroenterology 58:2003–2006, 2011. 11. Bachur RG, Hennelly K, Callahan MJ, et al: Diagnostic imaging and negative appen- dectomy rates in children: effects of age and gender. Pediatrics 129:877–884, 2012. 12. Miglioretti DL, Johnson E, Williams A, et al: The use of computed tomography in pediatrics and the associated radiation exposure and estimated cancer risk. JAMA Pediatr 167:700–707, 2013. 13. van Randen A, Laméris W, van Es HW, et al: A comparison of the accuracy of ultra- sound and computed tomography in common diagnoses causing acute abdominal pain. Eur Radiol 21:1535–1545, 2011. 14. Abo A, Shannon M, Taylor G, et al: The influence of body mass index on the accuracy of ultrasound and computed tomography in diagnosing appendicitis in children. Pediatr Emerg Care 27:731–736, 2011. 15. Mallin M, Craven P, Ockerse P, et al: Diagnosis of appendicitis by bedside ultrasound in the ED. Am J Emerg Med 33:430–432, 2015. 16. Elikashvili I, Tay ET, Tsung JW: The effect of point-of-care ultrasonography on emergency department length of stay and computed tomography utilization in chil- dren with suspected appendicitis. Acad Emerg Med 21:163–170, 2014. 17. Smith MP, Katz DS, Lalani T, et al: ACR Appropriateness Criteria® right lower quad- rant pain—suspected appendicitis. Ultrasound Q 31:85–91, 2015. 18. Kim K, Kim YH, Kim SY, et al: Low-dose abdominal CT for evaluating suspected appendicitis. N Engl J Med 366:1596–1605, 2012. 19. Gaetke-Udager K, Maturen KE, Hammer SG: Beyond acute appendicitis: imaging and pathologic spectrum of appendiceal pathology. Emerg Radiol 21:535–542, 2014. 20. Burke LMB, Bashir MR, Miller FH, et al: Magnetic resonance imaging of acute appendicitis in pregnancy: a 5-year multiinstitutional study. Am J Obstet Gynecol 213:693.e1–693.e6, 2015. 21. Expert Panel on MR Safety, Kanal E, Barkovich AJ, et al: ACR guidance document on MR safe practices: 2013. J Magn Reson Imaging 37:501–530, 2013. 22. Thirumoorthi AS, Fefferman NR, Ginsburg HB, et al: Managing radiation exposure in children—reexamining the role of ultrasound in the diagnosis of appendicitis. J Pediatr Surg 47:2268–2272, 2012. 23. Polites SF, Mohamed MI, Habermann EB, et al: A simple algorithm reduces com- puted tomography use in the diagnosis of appendicitis in children. Surgery 156:448– 454, 2014. 24. Bachur RG, Levy JA, Callahan MJ, et al: Effect of reduction in the use of computed tomography on clinical outcomes of appendicitis. JAMA Pediatr 169:755–760, 2015. 25. Aspelund G, Fingeret A, Gross E, et al: Ultrasonography/MRI versus CT for diagnos- ing appendicitis. Pediatrics 133:586–593, 2014. 26. Manterola C,Vial M, Moraga J, et al: Analgesia in patients with acute abdominal pain. Cochrane Database Syst Rev (1):CD005660, 2011. 27. Poonai N, Paskar D, Konrad S-L, et al: Opioid analgesia for acute abdominal pain in children: A systematic review and meta-analysis. Acad Emerg Med 21:1183–1192, 2014. 28. Solomkin JS, Mazuski JE, Bradley JS, et al: Diagnosis and management of compli- cated intra-abdominal infection in adults and children: guidelines by the Surgical Infection Society and the Infectious Diseases Society of America. Clin Infect Dis 50:133–164, 2010. 29. Salminen P, Paajanen H, Rautio T, et al: Antibiotic therapy vs appendectomy for treatment of uncomplicated acute appendicitis: the APPAC randomized clinical trial. JAMA 313:2340–2348, 2015. 30. Vons C, Barry C, Maitre S, et al: Amoxicillin plus clavulanic acid versus appendicec- tomy for treatment of acute uncomplicated appendicitis: an open-label, non- inferiority, randomised controlled trial. Lancet 377:1573–1579, 2011. 31. Shindoh J, Niwa H, Kawai K, et al: Predictive factors for negative outcomes in initial non-operative management of suspected appendicitis. J Gastrointest Surg 14:309– 314, 2010. 32. Seetahal SA, Bolorunduro OB, Sookdeo TC, et al: Negative appendectomy: a 10-year review of a nationally representative sample. Am J Surg 201:433–437, 2011. 33. Drake FT, Mottey NE, Farrokhi ET, et al: Time to appendectomy and risk of perfora- tion in acute appendicitis. JAMA Surg 149:837–844, 2014. 34. Ashdown HF, D’Souza N, Karim D, et al: Pain over speed bumps in diagnosis of acute appendicitis: diagnostic accuracy study. BMJ 345:e8012, 2012. 35. Laurell H, Hansson L-E, Gunnarsson U: Manifestations of acute appendicitis: a prospective study on acute abdominal pain. Dig Surg 30:198–206, 2013. CHAPTER 83: QUESTIONS & ANSWERS 83.1. What percentage of women with acute appendicitis have accompanying cervical motion tenderness (CMT)? A. 10% B. 15% C. 20% D. 25% E. 30% Answer: D. Prior to the advent of routine imaging of the appen- dix, as many as 25% of women with acute appendicitis were ini- tially misdiagnosed because of the presence of CMT. 83.2. Which of the following statements regarding ultrasonographic visualization of the appendix is true? A. A compressible appendix is a positive finding. B. An appendiceal diameter greater than 6 or 7 mm is a positive finding. C. The sensitivity of ultrasound for appendicitis is 94% to 98%. D. Ultrasonography has good reliability for detecting a retrocecal appendix. E. Ultrasonography compares favorably with computed tomography (CT) scanning for the detection of appendicitis. Answer: B. A noncompressible appendix with a diameter greater than 6 or 7 mm in a setting of clinical appendicitis is considered a positive finding. Ultrasound sensitivities are 75% to 90%. It is a less useful modality in the obese, those with peritoneal adhesions, and those with a retrocecal appendix. The sensitivity of helical CT scanning with rectal contrast approaches 98%, much higher than ultrasonography. 83.3. A 27-year-old G3P2 woman at 22 weeks of gestation presents with 2 days of right lower quadrant (RLQ) abdominal pain. It began midline and later became more pronounced in the RLQ. The physical examination was remarkable for RLQ tenderness without rebound. The gynecologic examination was negative except for a nontender gravid uterus, with good fetal movement by transabdominal ultrasound. Urinalysis showed 8 to 10 white blood cells (WBCs)/high-power field (HPF) and occasional bacteria. Complete blood count (CBC) showed a WBC count of 12,700/mm3 with 77% neutrophils. Hemoglobin level was 11 g/dL. RLQ ultrasound was limited, with no visualization of a normal or abnormal appendix, and transvaginal ultrasound did not show an obvious gynecologic or obstetric problem. Repeat examination showed continued RLQ tenderness. What is the most appropriate intervention? A. Administer cephalexin for urinary tract infection and schedule a 48-hour clinic recheck B. Admit for observation and serial examination Descargado para Francisco Medina Anderson (fz.medina@hotmail.com) en Organización de Estados Iberoamericanos - Remote Access de ClinicalKey.es por Elsevier en julio 07, 2017. Para uso personal exclusivamente. No se permiten otros usos sin autorización. Copyright ©2017. Elsevier Inc. Todos los derechos reservados.
  • 10. 1128.e2 PART III  Medicine and Surgery  |  SECTION Five  Gastrointestinal System C. Obtain surgical consultation for laparotomy D. Order a CT scan of the abdomen. E. Order a magnetic resonance imaging (MRI) scan Answer: E. MRI scanning for appendicitis may be helpful in preg- nant women, in whom the avoidance of radiation exposure is a significant consideration, and exploratory surgery carries addi- tional risks. 83.4. In men and children with classic symptoms and signs of appendicitis, what is the most appropriate initial intervention? A. Antibiotics and serial abdominal examinations B. CT scan of the abdomen C. MRI scan of the abdomen D. Surgery E. Ultrasonography Answer: E. In men and children with classic appendicitis, imaging adds little to the evaluation and only exposes patients to unneces- sary radiation. However, it has become less and less common for a patient with a history and examination concerning for appen- dicitis to undergo surgery without further imaging. Ultrasound is the most appropriate initial intervention because it uses no radia- tion and can often visualize and diagnose appendicitis without significant delay. Graded compression ultrasound for appendicitis is specific but lacks the sensitivity of CT scan so, if the appendix is not visualized, a discussion can be had with the general surgeon to determine if it is necessary to obtain further information (via CT or MRI). Descargado para Francisco Medina Anderson (fz.medina@hotmail.com) en Organización de Estados Iberoamericanos - Remote Access de ClinicalKey.es por Elsevier en julio 07, 2017. Para uso personal exclusivamente. No se permiten otros usos sin autorización. Copyright ©2017. Elsevier Inc. Todos los derechos reservados.